Qui si risolve LOGO
a

Menu

M

Chiudi

Studio di funzione: esercizi misti

Studio di funzione completo nel calcolo differenziale

Home » Studio di funzione: esercizi misti

Benvenuti nella nostra raccolta di esercizi sullo studio di funzione! In questo articolo presentiamo 24 esercizi su questo importante argomento, alcuni dei quali multipli, per un totale di 41 studi di funzione completamente svolti. Il materiale permette un approfondito studio di questo argomento, summa del programma svolto nei corsi di Analisi Matematica 1, ed è quindi particolarmente indicato per studenti universitari e appassionati.

Consigliamo il materiale teorico di riferimento:

Segnaliamo inoltre alcune raccolte di esercizi su argomenti affini:

Buona lettura!

 

Esercizi sullo studio di funzione: Sommario

Leggi...

Questo file contiene una raccolta ampia di esercizi dedicati allo studio di funzioni. Gli esercizi sono organizzati in ordine di difficoltà crescente, partendo da funzioni base fino ad arrivare ad espressioni più complesse. Ogni esercizio è seguito da una soluzione dettagliata. Questo materiale è ideale per studenti universitari che necessitano prepararsi per esami di analisi matematica.

 
 

Esercizi su studio di funzioni: Autori e revisori


 
 

Esercizi sullo studio di funzione

 
 

Esercizio 1  (\bigstar\largewhitestar\largewhitestar\largewhitestar\largewhitestar). Sia

    \[ f(x)=\frac{x^2-4x+3}{x-5}. \]

Tracciare il grafico della funzione dopo aver determinato: il dominio D, intersezione con gli assi, positività, continuità, asintoti, massimi e minimi, eventuali punti di non derivabilità e flessi. Una volta tracciato il grafico, esplicitare l’immagine di f.

Soluzione.

  1. Dominio Il dominio della funzione è dato da tutti i numeri reali eccetto il valore che annulla il denominatore, ovvero:

        \[ 		D = \mathbb{R} \setminus \{5\} 		\]

  2.  

  3. Intersezioni con gli assi

        \[\quad\]

    • Asse x: Poniamo f(x) = 0:

          \[ 			\frac{x^2 - 4x + 3}{x - 5} = 0 			\]

      Il numeratore si fattorizza come (x-3)(x-1), quindi le intersezioni con l’asse x sono A=(3,0) e B=(1,0).

    •  

    • Asse y: Calcoliamo f(0):

          \[ 			f(0) = \frac{0^2 - 4 \cdot 0 + 3}{0 - 5} = -\frac{3}{5} 			\]

      L’intersezione con l’asse y è pertanto C=\left(0, -\dfrac{3}{5}\right)

  4.  

  5. Studio del segno La funzione è positiva quando numeratore e denominatore hanno lo stesso segno:

        \[\quad\]

    • (x-3)(x-1) > 0 e (x-5) > 0, cioè per x > 5
    •  

    • (x-3)(x-1) < 0 e (x-5) < 0, cioè per 1 < x < 3

    Quindi f(x) > 0 per x \in (1,3) \cup (5,+\infty). Questo lo si può vedere anche graficamente con il seguente schema di studio del segno:

        \[\quad\]

        \[\quad\]

        \[\quad\]

    Rendered by QuickLaTeX.com

        \[\quad\]

        \[\quad\]

  6.  

  7. Continuità e derivabilità La funzione è continua e derivabile su tutto il suo dominio, essendo il rapporto di due polinomi.
  8.  

  9. Asintoti

        \[\quad\]

    • Asintoto orizzontale: Dato che

          \[ 			\lim_{x\to\pm\infty}f(x)=\lim_{x\to\pm\infty}\dfrac{x^2-4x+3}{x-5}=\lim_{x\to\pm\infty}\dfrac{x^2\left(1-\dfrac{4}{x}+\dfrac{3}{x^2}\right)}{x}=\lim_{x\to\pm\infty}x\left(1-\dfrac{4}{x}+\dfrac{3}{x^2}\right)=\pm\infty 			\]

      la funzione f(x) non ha alcun asintoto orizzontale.

    •  

    • Asintoto verticale: \lim_{x \to 5^\pm} f(x) = \pm\infty, quindi x = 5 è un asintoto verticale.
    •  

    • Asintoto obliquo: Calcoliamo:

          \[m = \lim_{x \to \pm\infty} \frac{f(x)}{x} = \lim_{x \to \pm\infty} \frac{x^2 - 4x + 3}{x(x-5)} = 1\]

          \[q = \lim_{x \to \pm\infty} [f(x) - mx] = \lim_{x \to \pm\infty} \left[\frac{x^2 - 4x + 3}{x-5} - x\right] = 1\]

      L’equazione dell’asintoto obliquo è quindi y = x + 1.

  10.  

  11. Massimi e minimi Calcoliamo la derivata prima:

        \[f'(x) = \frac{(2x-4)(x-5) - (x^2-4x+3)}{(x-5)^2} = \frac{x^2 - 10x + 17}{(x-5)^2}\]

    Poniamo f'(x) = 0:

        \[ 		x^2 - 10x + 17 = 0 \iff x_{1,2} = \frac{10 \pm \sqrt{100- 4 \cdot 17}}{2} = 5 \pm \sqrt{32}, 		\]

    pertanto le soluzioni sono x_{1,2} = 5 \pm 2\sqrt{2}. Il denominatore della derivata è positivo, perciò il segno di f' dipende solo dal numeratore:

        \[\quad\]

    • Nell’intervallo (5-2\sqrt 2,5 + 2 \sqrt 2) la derivata è negativa, dunque la funzione è decrescente.
    •  

    • In (-\infty,5-2\sqrt 2) \cup (5 + 2 \sqrt 2, \infty) la derivata è positiva, dunque la funzione è crescente.

    Di conseguenza, in x=5-2\sqrt 2 abbiamo un massimo relativo

        \[ 		f(5-2\sqrt{2}) = 6-4\sqrt{2}, 		\]

    mentre in x = 5 + 2\sqrt{2} abbiamo un minimo relativo:

        \[ 		f(5+2\sqrt{2}) = 6+4\sqrt{2}. 		\]

    Notiamo che questi sono punti di massimo e minimo relativi perché nella ricerca di asintoti orizzontali abbiamo mostrato che

        \[ 		\sup_{x\in D} f(x) = + \infty \quad \text{and} \quad \inf_{x \in D}f(x) =-\infty. 		\]

  12.  

  13. Flessi Calcoliamo la derivata seconda:

        \[f''(x) = \frac{16}{(x-5)^3}\]

    Questa non si annulla mai, quindi non ci sono flessi. Dallo studio del segno del denominatore, segue che la funzione è concava per x < 5 e convessa per x > 5.

  14.  

  15. Grafico

    \[\quad\]

    \[\quad\]

    \[\quad\]

Rendered by QuickLaTeX.com

    \[\quad\]

    \[\quad\]

Osservazioni sul grafico: Il grafico della funzione mostra:

    \[\quad\]

  • Due intersezioni con l’asse x in (1,0) e (3,0)
  •  

  • Un’intersezione con l’asse y in (0,-3/5)
  •  

  • Un asintoto verticale in x=5 (in rosso)
  •  

  • Un asintoto obliquo y = x + 1 (in verde)
  •  

  • Un massimo relativo prima dell’asintoto verticale M
  •  

  • Un minimo relativo dopo l’asintoto verticale m
  •  

  • La funzione è concava a sinistra dell’asintoto verticale e convessa a destra

 
 

Esercizio 2  (\bigstar\bigstar\largewhitestar\largewhitestar\largewhitestar). Sia

    \[ f(x)=\log \lvert x^2-8x+16\rvert. \]

Tracciare il grafico della funzione dopo aver determinato: il dominio D, intersezione con gli assi, positività, continuità, asintoti, massimi e minimi, eventuali punti di non derivabilità, flessi, concavità e convessità. Una volta tracciato il grafico, esplicitare l’immagine di f.

Soluzione.

  1. Dominio Il logaritmo richiede che l’argomento sia positivo. Osserviamo che:

        \[ 		x^2-8x+16 = (x-4)^2 \ge 0 		\]

    per ogni x \in \mathbb R. Di conseguenza, il modulo si può togliere

        \[| 		x^2-8x+16|=|(x-4)|^2=(x-4)^2. 		\]

    A questo punto, si deduce che D=\mathbb{R}\setminus \{4\} dato che l’argomento del logaritmo è sempre non-negativo e si annulla solo in x=4. Inoltre, usando le proprietà del logaritmo troviamo che

        \[ 		\log \left[(x-4)^2\right] = 2 \log|x-4|, 		\]

    espressione per f(x) che semplifica notevolmente i conti da fare per lo studio di funzione.

  2.  

  3. Intersezioni con gli assi

        \[\quad\]

    • Asse x: Poniamo f(x) = 0:

          \[2 \log |x-4| = 0 \iff |x-4| = 1\]

      utilizzando il fatto che \mathrm{e}^{\log y} = y. A questo punto possiamo risolvere sviluppando il modulo

          \[ 			|x-4|=1 \iff x - 4 = 1 \text{ oppure } x-4=-1 \iff x = 5 \text { oppure } x = 3	\]

      Le intersezioni con l’asse x sono pertanto A=(3,0) e B=(5,0).

    •  

    • Asse y: Calcoliamo f(0):

          \[f(0) = 2 \log(|0-4|) = 2 \log 4 = 4 \log 2 \]

      L’intersezione con l’asse y è C=(0, 4\log 2).

  4.  

  5. Positività La funzione è positiva quando l’argomento del logaritmo è maggiore di uno, ovvero |x-4| > 1. Togliendo il modulo si arriva alle disuguaglianze

        \[\begin{aligned} 		& x-4 > 1 & \implies x > 5 		\\ & x - 4 < - 1 & \implies x < 3 		\end{aligned}\]

    Quindi f(x) > 0 per x \in (-\infty, 3) \cup (5,+\infty) ed f(x)<0 per x \in (3,4)\cup(4,5).

  6.  

  7. Continuità e derivabilità La funzione è continua e derivabile con derivata continua nel suo dominio. Questo è facile da vedere perché abbiamo una composizione

        \[ 		\log(x) \circ |x-4| 		\]

    tra logaritmo e valore assoluto di un polinomio. Si noti che la funzione h(y)=|y| è derivabile ovunque tranne che in y=0; nel nostro caso, il punto che annulla l’argomento del logaritmo è escluso dal dominio (i.e., x\neq 4)

  8.  

  9. Asintoti

        \[\quad\]

    • Asintoti verticali1:

          \[\lim_{x \to 4^\pm} f(x) = \lim_{x \to 4^\pm} \log(x-4)^2 = \log 0^+ = -\infty\]

      Quindi x = 4 è un asintoto verticale.

    •  

    • Asintoti orizzontali:

          \[\lim_{x \to \pm\infty} f(x) = \lim_{x \to \pm\infty} \log(x-4)^2 = +\infty\]

      Non esistono asintoti orizzontali.

    •  

    • Asintoti obliqui:

          \[m = \lim_{x \to \pm\infty} \frac{f(x)}{x} = \lim_{x \to \pm\infty} \frac{\log(x-4)^2}{x} = 0\]

      Poiché m = 0, non esistono asintoti obliqui.

  10.  

  11. Massimi e minimi Calcoliamo la derivata prima:

        \[f'(x) = \frac{2}{x-4}\]

    La derivata non si annulla mai nel dominio, quindi non ci sono massimi o minimi relativi. Dallo studio dei limiti, \sup f(x) = +\infty e \inf f(x) = -\infty, quindi non ci sono massimi o minimi assoluti.

  12.  

  13. Flessi e convessità Calcoliamo la derivata seconda:

        \[f''(x) = -\frac{2}{(x-4)^2}\]

    f''(x) è sempre negativa nel dominio, quindi la funzione è sempre concava e non ha punti di flesso.

  14.  

  15. Grafico

    \[\quad\]

    \[\quad\]

    \[\quad\]

Rendered by QuickLaTeX.com

    \[\quad\]

    \[\quad\]

Osservazioni sul grafico: Il grafico della funzione mostra:

    \[\quad\]

  • Due intersezioni con l’asse x in (3,0) e (5,0)
  •  

  • Un’intersezione con l’asse y in (0, 4\log 2)
  •  

  • Un asintoto verticale in x=4 (in rosso)
  •  

  • La funzione tende a +\infty per x \to \pm\infty
  •  

  • La funzione è sempre concava
  •  

  • La funzione è simmetrica rispetto alla retta x=4

Si noti che la simmetria rispetto ad x=4 non è stata studiata nello svolgimento dell’esercizio, ma è banale dato che

    \[ 	f(x+4) = 2 \log |x|  	\]

e la funzione \log|x| è ovviamente pari e simmetrica rispetto l’origine. In particolare, lo studio di funzione si poteva svolgere limitandosi all’intervallo

    \[ 	D_+ = (4,+\infty) 	\]

per poi estendere i risultati ottenuti per simmetria (pari) rispetto l’origine.    


  1. Ricordiamo che x_0^+ ed x_0^- sono notazioni che si usano per indicare valori che stanno tendendo ad x_0 da destra e sinistra rispettivamente. Nel caso in oggetto, scrivere \log 0^+ è una forma compatta per intendere

        \[ 			\lim_{x \to 0^+} \log(x). 			\]


 
 

Esercizio 3  (\bigstar\largewhitestar\largewhitestar\largewhitestar\largewhitestar). Sia

    \[ f(x)=e^\frac{2x-1}{x^2}. \]

Tracciare il grafico della funzione dopo aver determinato: il dominio D, intersezione con gli assi, positività, continuità, asintoti, massimi e minimi, eventuali punti di non derivabilità, flessi, concavità e convessità. Una volta tracciato il grafico, esplicitare l’immagine di f.

Soluzione.

  1. Dominio Il dominio è D=\mathbb{R}\setminus \{0\} dato che il denominatore dell’esponente si annulla in x=0.
  2.  

  3. Intersezioni con gli assi

        \[\quad\]

    • Asse x: L’esponenziale è sempre positivo, quindi non ci sono intersezioni con l’asse x.
    •  

    • Asse y: x=0 non appartiene al dominio, quindi non c’è intersezione con l’asse y.
  4.  

  5. Positività L’esponenziale è una funzione sempre positiva, quindi f(x)>0 per ogni x\in D.
  6.  

  7. Continuità La funzione è continua su tutto il suo dominio, essendo la composizione di due funzioni continue.
  8.  

  9. Asintoti

        \[\quad\]

    • Asintoto verticale:

          \[\lim_{x \to 0^\pm} f(x) = \exp\left(\lim_{x \to 0^\pm} \frac{2x-1}{x^2}\right) = e^{-\infty} = 0\]

      Quindi x=0 non è un asintoto verticale, ma un punto di discontinuità eliminabile2.

    •  

    • Asintoto orizzontale:

          \[\lim_{x \to \pm\infty} f(x) = \exp\left(\lim_{x \to \pm\infty} \frac{2x-1}{x^2}\right) = e^0 = 1\]

      Quindi y=1 è un asintoto orizzontale.

  10.  

  11. Massimi e minimi Calcoliamo la derivata prima:

        \[f'(x) = \frac{e^{\frac{2x-1}{x^2}}(-2x+2)}{x^3}\]

    Studiamo il segno di f'(x):

        \[\quad\]

        \[\quad\]

        \[\quad\]

    Rendered by QuickLaTeX.com

        \[\quad\]

        \[\quad\]

    La derivata è strettamente positiva in (0,1). Quindi x=1 è un punto di massimo relativo (e assoluto) dove f(1)=e.

  12.  

  13. Flessi e convessità Calcoliamo la derivata seconda:

        \[f''(x) = \frac{e^{\frac{2x-1}{x^2}} (4x^3- 2x^2 - 8x +4)}{x^6}\]

    Le radici del numeratore sono \dfrac{1}{2}, \pm\sqrt{2}. Studiamo il segno di f''(x) tenendo conto che il denominatore è sempre positivo:

        \[\quad\]

        \[\quad\]

        \[\quad\]

    Rendered by QuickLaTeX.com

        \[\quad\]

        \[\quad\]

    La funzione è concava in (-\infty, -\sqrt{2}) \cup \left(\dfrac{1}{2},\sqrt{2}\right) e convessa altrove. I punti di flesso sono:

        \[\begin{aligned} 		B &\equiv \left(-\sqrt{2}, e^{\frac{-2\sqrt{2}-1}{2}}\right) \\ 		C &\equiv \left(\frac{1}{2}, 1\right) \\ 		D &\equiv \left(\sqrt{2}, e^{\frac{2\sqrt{2}-1}{2}}\right) 	\end{aligned}\]

  14.  

  15. Grafico

    \[\quad\]

    \[\quad\]

    \[\quad\]

Rendered by QuickLaTeX.com

    \[\quad\]

    \[\quad\]

Osservazioni conclusive: Il grafico della funzione mostra:

    \[\quad\]

  • Nessuna intersezione con gli assi
  •  

  • Un asintoto orizzontale y=1 per x \to \pm\infty (in verde)
  •  

  • Un punto di massimo assoluto in (1,e)
  •  

  • Tre punti di flesso
  •  

  • La funzione tende a 0 per x \to 0^\pm

   


  1. Diciamo che una funzione f(x) presenta una discontinuità eliminabile in un punto x = x_0 se esistono il limite \lim_{x \to x_0} f(x) finito e la funzione non è definita in x = x_0 o assume un valore diverso da \lim_{x \to x_0} f(x). In tal caso, è possibile eliminare la discontinuità ridefinendo la funzione come segue:

        \[ 			\tilde f(x) =\begin{cases} f(x) & \text{se } x \neq x_0 \\  \lim_{x \to x_0} f(x) & \text{se } x = x_0 \end{cases} 			\]

    In questo modo, la funzione diventa continua in x = x_0, ovvero la discontinuità è stata eliminata.


 
 

Esercizio 4  (\bigstar\bigstar\largewhitestar\largewhitestar\largewhitestar). Sia

    \[f(x)=e^{\tan(x)}.\]

Tracciare il grafico della funzione dopo aver determinato: il dominio D, intersezione con gli assi, positività, continuità, asintoti, massimi e minimi, eventuali punti di non derivabilità, flessi, concavità e convessità. Una volta tracciato il grafico, esplicitare l’immagine di f.

Soluzione.

  1. Dominio Il dominio di f corrisponde al dominio della funzione \tan(x), che risulta essere D=\{x \in \mathbb{R}: x \ne \dfrac{\pi}{2} + \pi k \text{ con } k \in \mathbb{Z}\}.
  2.  

  3. Intersezioni con gli assi

        \[\quad\]

    • Asse x: Non esistono intersezioni con l’asse x poiché e^x > 0 per ogni x \in \mathbb{R}.
    •  

    • Asse y: f(0) = e^{\tan(0)} = e^0 = 1. L’intersezione è data dal punto A = (0,1).
  4.  

  5. Positività La funzione f è sempre positiva su tutto il suo dominio, come menzionato in precedenza.
  6.  

  7. Continuità La funzione è continua in tutto il suo dominio D essendo composizione di funzioni continue (esponenziale ed arcotangente).
  8.  

  9. Periodicità e Asintoti

        \[\quad\]

    • Periodicità: La funzione ha periodo minimo pari a \pi. Infatti, si ha

          \[ 		f(x+\pi) = e^{\tan(x+\pi)} = e^{\tan(x)} = f(x). 		\]

    •  

    • Asintoti orizzontali e obliqui: Non esistono, poiché \lim_{x \to \pm\infty} f(x) non esiste. Infatti, dato che la funzione

          \[ 		h(x)= \tan(x) 		\]

      oscilla per x \to \pm \infty e, in particolare, ha asintoti verticali nei punti x_k = \pi/2 + k\pi, tende alternativamente a +\infty e -\infty ogni volta che x si avvicina a questi punti. Per dimostrare formalmente che il limite

          \[ 		\lim_{x \to +\infty} e^{\tan(x)} 		\]

      non esiste, è sufficiente fornire due sotto-successioni che portano a limiti diversi:

          \[\begin{aligned} 			& x_{n_k} = \frac{\pi}{2} + 2k\pi & \implies \lim_{k \to \infty} e^{\tan(x_{n_k})} = +\infty 			\\ & x_{m_k} = -\frac{\pi}{2} + 2k\pi &  \implies \lim_{k \to \infty} e^{\tan(x_{m_k})} = 0 		\end{aligned}\]

      Un discorso analogo si può fare per il limite \lim_{x \to - \infty}f(x).

    •  

    • Asintoti verticali: Calcoliamo i limiti per x \to \dfrac{\pi}{2}^\pm:

          \[\lim_{x \to \frac{\pi}{2}^+} f(x) = e^{-\infty} = 0\]

          \[\lim_{x \to \frac{\pi}{2}^-} f(x) = e^{+\infty} = +\infty\]

      Per la periodicità di f, x = \dfrac{\pi}{2} + k\pi è asintoto verticale per ogni k \in \mathbb{Z}.

  10.  

  11. Massimi e minimi Calcoliamo la derivata prima:

        \[f'(x) = e^{\tan(x)} \cdot \frac{1}{\cos^2(x)}\]

    Poiché f'(x) > 0 per ogni x \in D, la funzione f è monotona crescente su tutto il suo dominio. Dai limiti calcolati, abbiamo che \sup f = +\infty e \inf f = 0.

  12.  

  13. Flessi e convessità Calcoliamo la derivata seconda:

        \[f''(x) = e^{\tan(x)} \frac{1+2\sin(x)\cos(x)}{\cos^4(x)} = e^{\tan(x)} \frac{1+\sin(2x)}{\cos^4(x)}\]

    Il segno di f''(x) è determinato da 1+\sin(2x). Poiché \sin(2x) \ge -1 per ogni x, abbiamo che f''(x) \ge 0 per ogni x \in D. Di conseguenza, f è convessa su tutto il suo dominio D.

  14.  

  15. Grafico

    \[\quad\]

    \[\quad\]

    \[\quad\]

Rendered by QuickLaTeX.com

    \[\quad\]

    \[\quad\]

Osservazioni conclusive: Il grafico della funzione mostra:

    \[\quad\]

  • Periodicità con periodo \pi
  •  

  • Asintoti verticali in x = \dfrac{\pi}{2} + k\pi, k \in \mathbb{Z} (in verde)
  •  

  • Monotona crescente tra due asintoti consecutivi
  •  

  • Convessità su tutto il dominio
  •  

  • Intersezione con l’asse y nel punto (0,1)

 
 

Esercizio 5  (\bigstar\bigstar\bigstar\largewhitestar\largewhitestar). Sia

    \[f(x)=\sqrt{\log \lvert x-3 \rvert}. \]

Tracciare il grafico della funzione dopo aver determinato: il dominio D, intersezione con gli assi, positività, continuità, asintoti, massimi e minimi, eventuali punti di non derivabilità, flessi, concavità e convessità. Una volta tracciato il grafico, esplicitare l’immagine di f.

Soluzione.

  1. Dominio Il dominio della funzione è dato da:

        \[ 	\begin{cases} 		\log|x-3| \geq 0 \\ 		|x-3| > 0 	\end{cases} 	\quad \iff \quad 	\begin{cases} 		|x-3| \geq 1 \\ 		x \neq 3 	\end{cases} 	\quad \iff \quad 	x \leq 2 \quad \text{o} \quad x \geq 4 	\]

    Quindi, D = (-\infty, 2] \cup [4, +\infty).

  2.  

  3. Intersezioni con gli assi

        \[\quad\]

    • Asse x: Si ha

          \[ 		\sqrt{\log|x-3|} = 0 \iff \log|x-3| = 0 \iff |x-3| = 1, 		\]

      che ha soluzioni x=2 ed x=4; perciò, i punti di intersezione sono A=(2,0) e B=(4,0).

    •  

    • Asse y: dato che

          \[ 		f(0) = \sqrt{\log|-3|} = \sqrt{\log 3}, 		\]

      il punto di intersezione con l’asse y è C=(0,\sqrt{\log 3}).

  4.  

  5. Positività La funzione è sempre non negativa su tutto il suo dominio D.
  6.  

  7. Continuità La funzione è continua su tutto il dominio D essendo composizione di funzioni continue (modulo, logaritmo e radice).
  8.  

  9. Asintoti

        \[\quad\]

    • Asintoti verticali: Non esistono. In effetti, è facile vedere che

          \[\begin{aligned} 			& \lim_{x \to 2^-} f(x) = \sqrt{ \log 1^+ } = 0 			\\ & \lim_{x \to 4^+} f(x) = \sqrt{ \log 1^+ } = 0 		\end{aligned}\]

    •  

    • Asintoti orizzontali e obliqui: Usando la continuità della radice e del logaritmo per x in un intorno di \infty, passiamo il limite dentro:

          \[\lim_{x \to \pm\infty} f(x) = \sqrt{\log\lim_{x \to \pm\infty}|x-3|} = \sqrt{\log(+\infty)} = +\infty\]

      Non esistono asintoti orizzontali. In realtà, non ci sono neanche asintoti obliqui dato che

          \[ 		\lim_{x \to \pm \infty}  \frac{f(x)}{x} = 0 		\]

      è una banale conseguenza del fatto che il logaritmo va ad infinito più lentamente di ogni polinomio e, pertanto, anche della funzione h(x)=x.

  10.  

  11. Derivata prima e monotonia Si ha

        \[f'(x) = \frac{1}{2(x-3)\sqrt{\log|x-3|}}\]

    Studio del segno di f'(x):

        \[\quad\]

        \[\quad\]

        \[\quad\]

    Rendered by QuickLaTeX.com

        \[\quad\]

        \[\quad\]

    La funzione è decrescente in (-\infty,2) e crescente in (4,+\infty). Conseguentemente, i punti x=2 e x=4 sono di minimo assoluto.

  12.  

  13. Derivabilità in x=2 e x=4

        \[\quad\]

    • In x=2: \lim_{h \to 0^-} \dfrac{f(2+h)-f(2)}{h} = -\infty
    •  

    • In x=4: \lim_{h \to 0^+} \dfrac{f(4+h)-f(4)}{h} = +\infty

    La funzione non è derivabile in x=2 e x=4, e il grafico ha tangenti verticali in questi punti.

  14.  

  15. Derivata seconda e convessità

        \[f''(x) = \frac{-2\log|x-3| -1}{4(x-3)^2 \log^{\frac{3}{2}}|x-3|}\]

    Il denominatore è sempre positivo dove è definito, quindi per lo studio del segno di f''(x) consideriamo soltanto il numeratore:

        \[\quad\]

        \[\quad\]

        \[\quad\]

    Rendered by QuickLaTeX.com

        \[\quad\]

        \[\quad\]

    La funzione è concava su tutto il suo dominio dato che l’intervallo (2,4) va escluso.

  16.  

  17. Grafico

    \[\quad\]

    \[\quad\]

    \[\quad\]

Rendered by QuickLaTeX.com


 
 

Esercizio 6  (\bigstar\largewhitestar\largewhitestar\largewhitestar\largewhitestar). Sia

    \[ f(x)=\log \left(\frac{x-3}{x+4}\right). \]

Tracciare il grafico della funzione dopo aver determinato: il dominio D, intersezione con gli assi, positività, continuità, asintoti, massimi e minimi, eventuali punti di non derivabilità, flessi, concavità e convessità. Una volta tracciato il grafico, esplicitare l’immagine di f.

Soluzione.

  1. Dominio La condizione di esistenza del logaritmo richiede che il suo argomento sia positivo:

        \[\frac{x-3}{x+4} > 0\]

    Lo studio del segno porta al seguente diagramma:

        \[\quad\]

        \[\quad\]

        \[\quad\]

    Rendered by QuickLaTeX.com

        \[\quad\]

        \[\quad\]

    Quindi, il dominio è D = (-\infty, -4) \cup (3, +\infty).

  2.  

  3. Intersezioni con gli assi

        \[\quad\]

    • Asse x: Non esistono intersezioni con l’asse x, poiché l’equazione \log(x-3) = \log(x+4) non ha soluzioni come mostrato da

          \[ 		x-3 = x+4 \iff -3 = 4, 		\]

      che è ovviamente una uguaglianza falsa.

    •  

    • Asse y: Non esistono intersezioni con l’asse y, poiché x=0 non appartiene al dominio D.
  4.  

  5. Positività La funzione è positiva quando l’argomento del logaritmo è maggiore di uno:

        \[ 	\frac{x-3}{x+4} > 1 \quad \iff \quad \frac{-7}{x+4} > 0. 	\]

    Il numeratore è sempre negativo, perciò f(x) > 0 vale per x \in (-\infty, -4).

  6.  

  7. Continuità La funzione f è continua su tutto il suo dominio D essendo composizione di funzioni continue.
  8.  

  9. Asintoti

        \[\quad\]

    • Asintoti verticali:

          \[\lim_{x \to 3^+} f(x) = -\infty \quad \text{e} \quad \lim_{x \to -4^-} f(x) = +\infty\]

      Quindi, x=3 e x=-4 sono asintoti verticali.

    •  

    • Asintoto orizzontale:

          \[\lim_{x \to \pm\infty} f(x) = \log(1) = 0\]

      Quindi, y=0 è un asintoto orizzontale.

  10.  

  11. Derivata prima e monotonia Si ha

        \[ 	f'(x) = \frac{7}{(x-3)(x+4)}. 	\]

    Il segno di f'(x) è determinato da (x-3)(x+4), che è positivo in (-\infty, -4) \cup (3, +\infty), coincidente con il dominio D. In particolare:

        \[\quad\]

    • f è crescente su tutto il suo dominio;
    •  

    • f non ammette massimi o minimi relativi.
  12.  

  13. Derivata seconda e convessità Si ha

        \[ 	f''(x) = \frac{-7(2x+1)}{(x-3)^2(x+4)^2} 	\]

    Il denominatore è prodotto di termini positivi, quindi il segno di f''(x) è determinato dal numeratore -7(2x+1). In particolare, otteniamo:

        \[\quad\]

    • f''(x) > 0 per x < -\dfrac{1}{2}
    •  

    • f''(x) < 0 per x > -\dfrac{1}{2}

    Tuttavia, x = - 1/2 non appartiene a D, quindi non ci sono flessi. Intersecando gli intervalli ottenuti con il dominio, deduciamo che la funzione è convessa per x < -4 e concava per x > 3.

  14.  

  15. Grafico

    \[\quad\]

    \[\quad\]

    \[\quad\]

Rendered by QuickLaTeX.com


 
 

Esercizio 7  (\bigstar\bigstar\largewhitestar\largewhitestar\largewhitestar). Sia

    \[ f(x)=\sqrt{\frac{x^3-1}{x}}. \]

Tracciare il grafico della funzione dopo aver determinato: il dominio D, intersezione con gli assi, positività, continuità, asintoti, massimi e minimi, eventuali punti di non derivabilità, flessi, concavità e convessità. Una volta tracciato il grafico, esplicitare l’immagine di f.

Soluzione.

  1. Dominio Il dominio è dato dall’argomento della radice non-negativo \left(\dfrac{x^3-1}{x} \geq 0\right) ed il denominatore diverso da zero (x \neq 0). Lo studio del segno porta al seguente diagramma:

        \[\quad\]

        \[\quad\]

        \[\quad\]

    Rendered by QuickLaTeX.com

        \[\quad\]

        \[\quad\]

    Quindi, il dominio è D = (-\infty, 0) \cup [1, +\infty).

  2.  

  3. Intersezioni con gli assi

        \[\quad\]

    • Asse x: Si noti che f(x) = 0 se e solo se x^3 - 1=0, che ha come unica soluzione x=1. Di conseguenza, A=(1,0) è l’unico punto di intersezione con l’asse x.
    •  

    • Asse y: Non ci sono intersezioni con l’asse y poiché 0 \notin D.
  4.  

  5. Positività La funzione è sempre non negativa su tutto il suo dominio D.
  6.  

  7. Continuità La funzione è continua su tutto il dominio D essendo composizione di funzioni continue.
  8.  

  9. Asintoti

        \[\quad\]

    • Asintoto verticale: \lim_{x \to 0^-} f(x) = +\infty, quindi x=0 è un asintoto verticale.
    •  

    • Asintoti orizzontali: Si osserva facilmente che

          \[ 		\lim_{x \to \pm \infty} \sqrt{\frac{x^3-1}{x}} = \lim_{x \to \pm \infty} \sqrt{x^2} = + \infty, 		\]

      quindi non esistono asintoti orizzontali.

    •  

    • Asintoti obliqui: Iniziamo con il caso + \infty. Il coefficiente angolare è

          \[ 		m_+ = \lim_{x \to + \infty} \frac{f(x)}{x}  = \lim_{x \to + \infty} \frac{|x|}{x}  = 1, 		\]

      dove abbiamo usato l’identità \sqrt{x^2} = |x|. Segue che

          \[ 		q_+ = \lim_{x \to + \infty} f(x) - m_+ x = \lim_{x \to + \infty} |x|-x = 0, 		\]

      quindi y = x è asintoto obliquo per x \to + \infty. Nel caso - \infty si possono seguire gli stessi conti, osservando che

          \[ 		m_- = \lim_{x \to - \infty} \frac{|x|}{x}  = -1 \quad \text{e} \quad q_- = 0, 		\]

      da cui segue che y=-x è asintoto obliquo per x \to - \infty.

  10.  

  11. Derivata prima e monotonia

        \[f'(x) = \frac{2x^3+1}{2x^2} \sqrt{\frac{x}{x^3-1}}\]

    Il termine sotto radice e 2x^2 sono entrambi termini non-negativi, perciò il segno di f' corrisponde a quello di 2x^3+1. Si ha

        \[ 	2x^3+1 > 0 \iff x^3 > - \frac12 \iff x > -\frac{1}{2^{1/3}}. 	\]

    Dunque, intersecando con il dominio, la funzione risulta decrescente per x < -\dfrac{1}{2^{1/3}} e crescente per -\dfrac{1}{2^{1/3}} < x < 0 e x > 1. Il punto di minimo relativo è x = -\dfrac{1}{2^{1/3}} con f\left(-\dfrac{1}{2^{1/3}}\right) = \dfrac{\sqrt{3}}{\sqrt[3]{2}}. Poiché

        \[ 	f(1) = 0, 	\]

    segue che (1,0) è il punto di minimo assoluto.

  12.  

  13. Derivata seconda e convessità

        \[f''(x) = \sqrt{\frac{x^3-1}{x}}\left(\frac{-12x^3+3}{4x^2(x^3-1)^2}\right)\]

    Il segno di f''(x) è determinato da -12x^3+3. La funzione è convessa per x > \dfrac{1}{2^{2/3}} e concava per x < \dfrac{1}{2^{2/3}}. Poiché \dfrac{1}{2^{2/3}} \notin D, non ci sono punti di flesso.

  14.  

  15. Grafico

    \[\quad\]

    \[\quad\]

    \[\quad\]

Rendered by QuickLaTeX.com

    \[\quad\]

    \[\quad\]

Osservazioni conclusive:

    \[\quad\]

  • Un asintoto verticale in x=0 (in rosso)
  •  

  • Asintoti obliqui y=x per x \to +\infty e y=-x per x \to -\infty (in verde)
  •  

  • Un punto di minimo relativo per x = - 2^{-1/3} (m_r)
  •  

  • Un punto di minimo assoluto in m=(1,0)
  •  

  • Concavità che cambia, ma senza punti di flesso nel dominio

 
 

Esercizio 8  (\bigstar\bigstar\largewhitestar\largewhitestar\largewhitestar). Sia

    \[ f(x)=e^{3x} + e^{\lvert x \rvert}. \]

Tracciare il grafico della funzione dopo aver determinato: il dominio D, intersezione con gli assi, positività, continuità, asintoti, massimi e minimi, eventuali punti di non derivabilità, flessi, concavità e convessità. Una volta tracciato il grafico, esplicitare l’immagine di f.

Soluzione.

  1. Dominio Il dominio è D=\mathbb{R}, poiché sia il valore assoluto che l’esponenziale sono definiti per ogni numero reale.
  2.  

  3. Intersezioni con gli assi

        \[\quad\]

    • Asse x: Non ci sono intersezioni con l’asse x poiché f(x) > 0 per ogni x \in \mathbb{R} essendo somma di funzioni strettamente positive.
    •  

    • Asse y: f(0) = e^0 + e^0 = 2. Quindi il punto di intersezione è A=(0,2).
  4.  

  5. Positività La funzione è sempre positiva su tutto il suo dominio, come menzionato nel punto precedente.
  6.  

  7. Continuità La funzione è continua su tutto \mathbb{R} in quanto somma di composizioni di funzioni continue.
  8.  

  9. Asintoti

        \[\quad\]

    • Asintoti verticali: Non esistono.
    •  

    • Asintoti orizzontali: Non esistono, poiché \lim_{x \to \pm\infty} f(x) = +\infty.
    •  

    • Asintoti obliqui: Non esistono, poiché \lim_{x \to \pm\infty} \dfrac{f(x)}{x} = +\infty.
  10.  

  11. Derivata prima e monotonia Sviluppando il modulo |x|=x per x > 0 e |x|=-x per x < 0, otteniamo:

        \[f'(x) = \begin{cases} 		3e^{3x}+e^x & \text{se } x > 0 \\ 		3e^{3x}-e^{-x} & \text{se } x < 0 	\end{cases}\]

    Per x > 0 abbiamo la somma di due termini positivi, quindi risulta sempre strettamente maggiore di zero. Per x < 0, invece:

        \[ 	3e^{3x} - e^{-x} = 0 \iff 3e^{4x} = 1 \iff e^{4x} = \frac13, 	\]

    e passando alla funzione inversa (ovvero il logaritmo) si ottiene

        \[ 	4x = \log \frac13 = - \log 3, 	\]

    che ha come soluzione x = - \dfrac14 \log 3. Perciò la funzione è decrescente per x < -\dfrac{\log 3}{4} e crescente per x > -\dfrac{\log 3}{4}. Il punto di minimo è x = -\dfrac{\log 3}{4} con

        \[ 	f(-\frac{\log 3}{4}) = \frac{4}{\sqrt[4]{3^3}}. 	\]

    Si verifica facilmente che questo è anche un punto di minimo assoluto.

  12.  

  13. Derivabilità in x = 0 Calcoliamo il limite del rapporto incrementale in x = 0. Da destra, si ha

        \[ 	\lim_{h \to 0^+} \frac{f(h)-f(0)}{h} = 4. 	\]

    mentre da sinistra

        \[ 	\lim_{h \to 0^-} \frac{f(h)-f(0)}{h} = 2 	\]

    Poiché i limiti destro e sinistro sono diversi, la funzione non è derivabile in x = 0, che risulta essere un punto angoloso (dato che i limiti esistono pur essendo diversi).

  14.  

  15. Derivata seconda e convessità

        \[f''(x) = 9e^{3x} + e^{|x|}\]

    Poiché f''(x) > 0 per ogni x \in \mathbb{R}, la funzione è convessa su tutto il suo dominio.

  16.  

  17. Grafico

    \[\quad\]

    \[\quad\]

    \[\quad\]

Rendered by QuickLaTeX.com


 
 

Esercizio 9  (\bigstar\largewhitestar\largewhitestar\largewhitestar\largewhitestar). Sia

    \[ f(x)=\arctan(x^2-1). \]

Tracciare il grafico della funzione dopo aver determinato: il dominio D, intersezione con gli assi, positività, continuità, asintoti, massimi e minimi, eventuali punti di non derivabilità, flessi, concavità e convessità. Una volta tracciato il grafico, esplicitare l’immagine di f.

Soluzione.

  1. Dominio Il dominio è D=\mathbb{R}, poiché la funzione arcotangente è definita per ogni numero reale.
  2.  

  3. Intersezioni con gli assi

        \[\quad\]

    • Asse x: L’arcotangente si annulla solo quando l’argomento è zero, perciò:

          \[ 		x^2 - 1 = 0, 		\]

      che ha come soluzioni x = \pm 1. Dunque, le intersezioni sono A=(1,0) e B=(-1,0).

    •  

    • Asse y: f(0) = \arctan(-1) = -\pi/4, perciò il punto di intersezione è C=(0,-\pi/4).
  4.  

  5. Simmetria La funzione è pari dato che soddisfa la relazione

        \[ 	f(x) = f(-x). 	\]

    Di conseguenza, possiamo limitarci a studiarne le proprietà nella semiretta positiva D_+ := [0,+\infty).

  6.  

  7. Positività La funzione è positiva quando x^2 - 1 > 0, cioè per x \in (1, +\infty).
  8.  

  9. Continuità La funzione f è continua su tutto il suo dominio D_+ essendo composizione di funzioni continue.
  10.  

  11. Asintoti

        \[\quad\]

    • Asintoti verticali: Non esistono poiché non sono presenti punti di discontinuità.
    •  

    • Asintoto orizzontale: Basta ricordare che l’arcotangente soddisfa

          \[ 		\lim_{x \to+ \infty} \arctan(x) = \frac\pi2 		\]

      per concludere che y = \pi/2 è un asintoto orizzontale per x \to \pm\infty (per simmetria).

    •  

    • Asintoti obliqui: Non esistono.
  12.  

  13. Derivata prima e monotonia La derivata prima è data da

        \[f'(x) = \frac{2x}{x^4-2x^2+2}\]

    Il denominatore si può scrivere come

        \[ 	x^4-2x^2+2 = 1 + (-1 + x^2)^2 	\]

    ed è sempre positivo, perciò il segno di f'(x) dipende solo dal numeratore x. La funzione è decrescente per x < 0 e crescente per x > 0, e questo è compatibile con il fatto che

        \[ 	f(x) \text{ pari } \implies f'(x) \text{ dispari} 	\]

    Il punto di minimo (assoluto) è x = 0, corrispondente al punto C = (0,-\pi/4). Non esistono massimi assoluti poiché \sup f = \pi/2.

  14.  

  15. Derivata seconda e convessità

        \[f''(x) = \frac{-6x^4 + 4x^2 + 4}{(x^4-2x^2+2)^2}\]

    Il denominatore è sempre positivo, perciò ci concentriamo sullo studio del segno del numeratore. Sostituendo t = x^2 si ha

        \[-6t^2 + 4t + 4 = 0\]

    che ha come soluzioni

        \[t_{1,2} = \frac{1 \pm \sqrt{7}}{3}.\]

    Dato che t = x^2, la soluzione negativa si può scartare. Considerando solo la positiva e risolvendo per x, si trova

        \[x_{1,2} = \pm \sqrt{\frac{1 + \sqrt{7}}{3}}.\]

    A questo punto, il segno di f''(x) si comporta come segue:

        \[\quad\]

        \[\quad\]

        \[\quad\]

    Rendered by QuickLaTeX.com

        \[\quad\]

        \[\quad\]

    La funzione è convessa per x < -\sqrt{\dfrac{1+\sqrt{7}}{3}} e x > \sqrt{\dfrac{1+\sqrt{7}}{3}}, e concava per -\sqrt{\dfrac{1+\sqrt{7}}{3}} < x < \sqrt{\dfrac{1+\sqrt{7}}{3}}.

    I punti di flesso sono:

        \[x = \pm \sqrt{\frac{1+\sqrt{7}}{3}} \quad \text{con} \quad y = \arctan\left(\frac{\sqrt{7}-1}{2}\right)\]

  16.  

  17. Grafico

    \[\quad\]

    \[\quad\]

    \[\quad\]

Rendered by QuickLaTeX.com


 
 

Esercizio 10  (\bigstar\bigstar\bigstar\largewhitestar\largewhitestar). Sia

    \[ 	f(x)=\begin{cases} 	\sin(x)+x & x>0\\ 	x&x\le 0 \end{cases}\]

Tracciare il grafico della funzione dopo aver determinato: il dominio D, intersezione con gli assi, positività, continuità, asintoti, massimi e minimi, eventuali punti di non derivabilità, flessi, concavità e convessità. Una volta tracciato il grafico, esplicitare l’immagine di f.

Soluzione.

  1. Dominio Il dominio è D=\mathbb{R}, poiché entrambe le parti della funzione sono definite per ogni numero reale.
  2.  

  3. Intersezione con gli assi La funzione interseca gli assi cartesiani solo nell’origine O=(0,0). Infatti, è facile verificare che

        \[ 	\sin x + x = 0 \iff x = - \sin x, 	\]

    e questa ha come unica soluzione x = 0, che però appartiene all’altro ramo della funzione a tratti.

  4.  

  5. Positività

        \[\quad\]

    • Per x > 0: f(x) = \sin(x) + x > 0 sempre, poiché \sin(x) > -x per x > 0. Si può verificare banalmente osservando che

          \[ 		f(0)=0 \quad \text{e} \quad f'(x) = \cos (x) + 1, 		\]

      quindi la funzione è strettamente crescente in \cup_{k \in \mathbb N}(0, 2k \pi).

    •  

    • Per x \leq 0: f(x) = x \leq 0.

        \[\quad\]

        \[\quad\]

        \[\quad\]

    Rendered by QuickLaTeX.com

        \[\quad\]

        \[\quad\]

  6.  

  7. Continuità La funzione è continua in x = 0 poiché:

        \[\lim_{x \to 0^+} (\sin x + x) = \lim_{x \to 0^-} x = f(0) = 0\]

    Di conseguenza, f è continua su tutto il suo dominio.

  8.  

  9. Asintoti La funzione non ammette asintoti orizzontali dato che

        \[ \begin{aligned} 		& \lim_{x \to + \infty} f(x) = \lim_{x \to + \infty} (\sin x + x) = + \infty, 		\\ & \lim_{x \to - \infty} f(x) = \lim_{x \to - \infty} x = - \infty. 	\end{aligned}\]

    Il primo limite è una conseguenza banale del fatto che la funzione \sin x è limitata, perciò

        \[ 	x-1 \le x + \sin x  \le x +1, 	\]

    e prendendo il limite per x \to + \infty il teorema del confronto ci permette di concludere.

    Non sono neanche presenti asintoti verticali dato che f è definita su tutto l’insieme dei numeri reali. Per quanto riguarda gli obliqui:

        \[\quad\]

    • Per x \to + \infty si ha coefficiente angolare

          \[ 		m_+ = \lim_{x \to + \infty} \frac{f(x)}{x} = \lim_{x \to + \infty} \frac{\sin x + x}{x} = 1, 		\]

      per cui

          \[ 		q_+ = \lim_{x \to + \infty} f(x) - m_+ x =  \lim_{x \to + \infty} \sin x, 		 \]

      che non esiste. In particolare, non ci sono asintoti obliqui a destra.

    •  

    • Per x \to - \infty si ha coefficiente angolare

          \[ 		m_- = \lim_{x \to - \infty} \frac{f(x)}{x} = \lim_{x \to - \infty} \frac{x}{x} = 1, 		\]

      per cui

          \[ 		q_ = \lim_{x \to - \infty} f(x) - m_- x =  \lim_{x \to + \infty} (x - x) = 0, 		\]

      perciò y = x è (banalmente) un asintoto obliquo per x \to - \infty.

  10.  

  11. Derivata prima e monotonia

        \[f'(x) = \begin{cases} 		\cos(x) + 1 & \text{se } x > 0 \\ 		1 & \text{se } x < 0 	\end{cases}\]

    f ha un punto angoloso in x = 0 poiché i limiti da destra e sinistra esistono ma non coincidono:

        \[ 	\lim_{x \to 0^+} f'(x) = \cos 0 + 1 = 2 \neq 1 = 1 = \lim_{x \to 0^-} f'(x) 	\]

    La funzione è crescente su tutto il suo dominio poiché f'(x) > 0 per ogni x \neq 0.

  12.  

  13. Derivata seconda e convessità

        \[f''(x) = \begin{cases} 		-\sin(x) & \text{se } x > 0 \\ 		0 & \text{se } x \leq 0 	\end{cases}\]

        \[\quad\]

    • Per x \leq 0: la funzione non è né concava né convessa dato che coincide con una retta (y = x, nello specifico).
    •  

    • Per x > 0: la funzione è concava nell’intervallo (0, \pi], convessa nell’intervallo [\pi, 2\pi], e questo comportamento si ripete con periodicità 2\pi.

    I punti di flesso sono x_k = k\pi con k \in \mathbb{N}, dove f(k\pi) = k\pi.

  14.  

  15. Grafico

    \[\quad\]

    \[\quad\]

    \[\quad\]

Rendered by QuickLaTeX.com

    \[\quad\]

    \[\quad\]

Osservazioni conclusive:

    \[\quad\]

  • Una parte lineare per x \leq 0
  •  

  • Una parte sinusoidale sovrapposta a una retta per x > 0
  •  

  • Un punto angoloso nell’origine
  •  

  • Crescita monotona su tutto il dominio

 

Scarica gli esercizi svolti

Ottieni il documento contenente 41 esercizi svolti sullo studio di funzione.

 
 

Esercizio 11  (\bigstar\bigstar\largewhitestar\largewhitestar\largewhitestar). Sia

    \[ f(x)=e^{-\frac{1}{x^2}}. \]

Tracciare il grafico della funzione dopo aver determinato: il dominio D, intersezione con gli assi, positività, continuità, asintoti, massimi e minimi, eventuali punti di non derivabilità, flessi, concavità e convessità. Una volta tracciato il grafico, esplicitare l’immagine di f.

Soluzione.

  1. Dominio e simmetrie Il dominio è D=\mathbb{R}\setminus\{0\}, poiché la funzione 1/x^2 non è definita per x=0. Inoltre, è facile vedere che

        \[ 	f(-x) = e^{- 1/(-x)^2}  = e^{- 1/x^2} = f(x), 	\]

    quindi la funzione è pari e, di conseguenza, possiamo limitarci a studiarne il comportamento nella semiretta positiva D_+ = (0,\infty).

  2.  

  3. Intersezioni con gli assi

        \[\quad\]

    • Asse x: Non esistono intersezioni con l’asse x poiché f(x) > 0 per ogni x \in D_+.
    •  

    • Asse y: Non esistono intersezioni con l’asse y poiché x=0 non appartiene al dominio.
  4.  

  5. Positività La funzione è positiva su tutto il suo dominio D_+ essendo un esponenziale.
  6.  

  7. Continuità La funzione f è continua su tutto il suo dominio D_+ in quanto composizione di funzioni continue. In effetti, h(x)=1/x^2 è continua quando x \neq 0.
  8.  

  9. Asintoti E’ facile verificare che:

        \[\quad\]

    • Asintoto orizzontale: Per x \to+ \infty si ha

          \[\lim_{x \to + \infty} e^{-\frac{1}{x^2}} = e^0 = 1, 		\]

      perciò y = 1 è un asintoto orizzontale sia per + \infty che (simmetria) per -\infty.

    •  

    • Asintoto verticale: Si verifica nel punto di discontinuità x=0. Per simmetria, i limiti da destra e sinistra coincidono e si ha:

          \[ 		\lim_{x \to 0^+} e^{-\frac{1}{x^2}} = e^{-\infty} = 0, 		\]

      perciò non ci sono asintoti verticali, ma una discontinuità eliminabile.

  10.  

  11. Derivata prima e monotonia

        \[f'(x) = \frac{2}{x^3} e^{-\frac{1}{x^2}}\]

    La funzione esponenziale è positiva, quindi il segno di f'(x) dipende solo da x^3:

        \[\quad\]

    • f'(x) < 0 per x < 0 (funzione decrescente)
    •  

    • f'(x) > 0 per x > 0 (funzione crescente)

    Questo comportamento si poteva dedurre anche dal fatto che, come menzionato in un esercizio precedente,

        \[ 	f \text { pari } \implies f' \text{ dispari } \implies f'' \text { pari } \implies \cdots 	\]

    Inoltre, si verifica facilmente che \inf f = 0 è ottenuto per x \to 0 e \sup f = 1 per x \to + \infty. Non ci sono punti di massimo o minimo relativo/assoluto.

  12.  

  13. Derivata seconda e convessità Utilizzando la formula per la derivata di un prodotto, si trova:

        \[f''(x) = \frac{4-6x^2}{x^6} e^{-\frac{1}{x^2}}\]

    In questo caso, l’esponenziale e il denominatore sono positivi, perciò il segno di f''(x) dipende dal polinomio di secondo grado 4-6x^2:

        \[\quad\]

        \[\quad\]

        \[\quad\]

    Rendered by QuickLaTeX.com

        \[\quad\]

        \[\quad\]

    La funzione risulta dunque essere

        \[\quad\]

    • convessa in \left(-\sqrt{\dfrac{2}{3}}, \sqrt{\dfrac{2}{3}}\right) \setminus \{0\}
    •  

    • concava in \left(-\infty, -\sqrt{\dfrac{2}{3}}) \cup (\sqrt{\dfrac{2}{3}}, +\infty\right)

    ed i corrispondenti punti di flesso sono

        \[A=\left(-\sqrt{\frac{2}{3}}, e^{-\frac{3}{2}}\right) \quad \text{e} \quad B=\left(\sqrt{\frac{2}{3}}, e^{-\frac{3}{2}}\right)\]

  14.  

  15. Grafico

    \[\quad\]

    \[\quad\]

    \[\quad\]

Rendered by QuickLaTeX.com


 
 

Esercizio 12  (\bigstar\largewhitestar\largewhitestar\largewhitestar\largewhitestar). Sia

    \[ f(x)=\sqrt{x^2-3x+7}. \]

Tracciare il grafico della funzione dopo aver determinato: il dominio D, intersezione con gli assi, positività, continuità, asintoti, massimi e minimi, eventuali punti di non derivabilità, flessi, concavità e convessità. Una volta tracciato il grafico, esplicitare l’immagine di f.

Soluzione.

  1. Dominio Il dominio è determinato dalla condizione x^2-3x+7 \geq 0, ovvero argomento della radice non negativo. Risolvendo l’equazione associata si trova

        \[x_{1,2} = \frac{3 \pm \sqrt{9-4\cdot 7}}{2} = \frac{3 \pm \sqrt{-19}}{2},\]

    e poiché il discriminante è negativo, il polinomio x^2-3x+7 risulta essere sempre positivo. Quindi, il dominio è D=\mathbb{R}.

  2.  

  3. Intersezioni con gli assi

        \[\quad\]

    • Asse x: Non esistono intersezioni con l’asse x come verificato nel conto precedente.
    •  

    • Asse y: f(0) = \sqrt{7}, quindi il punto di intersezione è A=(0,\sqrt{7}).
  4.  

  5. Positività La funzione è sempre positiva su tutto il suo dominio \mathbb{R}. Infatti, ricordiamo che la radice quadrata soddisfa

        \[ 	\sqrt{ h(x) } \ge 0 \qquad \text{per ogni $x$ nel dominio}. 	\]

  6.  

  7. Continuità La funzione f è continua su tutto \mathbb{R} in quanto composizione di funzioni continue.
  8.  

  9. Asintoti

        \[\quad\]

    • Asintoti verticali: Non ci sono punti di discontinuità, pertanto non ci sono asintoti verticali.
    •  

    • Asintoti orizzontali: Non esistono, poiché

          \[ 		\lim_{x \to \pm\infty} f(x) = +\infty. 		\]

    •  

    • Asintoti obliqui:

      – Per x \to +\infty, il coefficiente angolare è

          \[ 			m_+ = \lim_{x \to + \infty} \frac{f(x)}{x} = \lim_{x \to + \infty} \frac{\sqrt{x^2}}{x} = \lim_{x \to + \infty} \frac{|x|}{x} = 1, 			\]

      dove si è usata la solita identità \sqrt{x^2}=|x|. Ne segue che

          \[ 			q_+ =  \lim_{x \to + \infty} f(x) - m_+ x = \lim_{x \to + \infty} \sqrt{x^2 - 3x + 7} - x = - \frac32, 			\]

      come si può mostrare facilmente razionalizzando. Perciò y = x - 3/2 è un asintoto obliquo nel limite per x \to + \infty.

      – Per x \to -\infty, il coefficiente angolare è

          \[ 			m_- = \lim_{x \to - \infty} \frac{f(x)}{x} = \lim_{x \to - \infty} \frac{|x|}{x} = -1, 			\]

      da cui segue che

          \[ 			q_- =  \lim_{x \to - \infty} f(x) - m_- x = \lim_{x \to - \infty} \sqrt{x^2 - 3x + 7} + x = \frac32, 			\]

      come si può mostrare facilmente razionalizzando. Perciò y = -x + 3/2 è un asintoto obliquo nel limite per x \to - \infty.

  10.  

  11. Derivata prima e monotonia

        \[f'(x) = \frac{2x-3}{2\sqrt{x^2-3x+7}}\]

    Il segno di f'(x) dipende solo dal numeratore 2x-3, perciò

        \[\quad\]

    • f'(x) < 0 per x < \dfrac{3}{2} (funzione decrescente)
    •  

    • f'(x) > 0 per x > \dfrac{3}{2} (funzione crescente)

    Il punto di minimo relativo (e assoluto) è x = \dfrac{3}{2} con f\left(\dfrac{3}{2}\right) = \dfrac{\sqrt{19}}{2}.

  12.  

  13. Derivata seconda e convessità

        \[f''(x) = \frac{19}{4(\sqrt{x^2-3x+7})^3}\]

    Poiché f''(x) > 0 per ogni x \in \mathbb{R}, la funzione è sempre convessa e non ha punti di flesso.

  14.  

  15. Grafico

    \[\quad\]

Rendered by QuickLaTeX.com


 
 

Esercizio 13  (\bigstar\largewhitestar\largewhitestar\largewhitestar\largewhitestar). Sia

    \[ f(x)=\frac{4x-12}{(x-2)^2}. \]

Tracciare il grafico della funzione dopo aver determinato: il dominio D, intersezione con gli assi, positività, continuità, asintoti, massimi e minimi, eventuali punti di non derivabilità, flessi, concavità e convessità. Una volta tracciato il grafico, esplicitare l’immagine di f.

Soluzione.

  1. Dominio Il dominio è determinato dalla condizione (x-2)^2 \neq 0, quindi:

        \[ D = (-\infty, 2) \cup (2, +\infty)\]

  2.  

  3. Simmetria La funzione non presenta simmetrie poiché:

        \[f(-x) = \frac{4(-x)-12}{(-x-2)^2} = -\frac{4x+12}{(x+2)^2} \neq \pm f(x)\]

  4.  

  5. Intersezioni con gli assi

        \[\quad\]

    • Asse y: f(0) = -3, quindi A=(0,-3) è il punto di intersezione
    •  

    • Asse x: f(x) = 0 se e solo se si annulla il denominatore, ovvero 4x - 12 = 0. Di conseguenza, il punto B=(3,0) è l’unico di intersezione con l’asse x.
  6.  

  7. Positività La funzione è positiva per x \in (3, +\infty) in quanto il denominatore non gioca alcun ruolo nel determinare il segno.
  8.  

  9. Asintoti

        \[\quad\]

    • Asintoto orizzontale: Si vede che

          \[ 			\lim_{x \to \pm \infty} f(x) = 0 			\]

      dato che il denominatore va come x^2 ed il numeratore come x. Pertanto, la retta y=0 è un asintoto orizzontale per x \to \pm \infty.

    •  

    • Asintoto verticale: Verifichiamo nell’unico punto di discontinuità, ovvero x = 2. Si ha

          \[ 			\lim_{x \to 2^\pm} f(x) = - \frac4{0^+} = - \infty, 			\]

      quindi x = 2 è un asintoto verticale.

  10.  

  11. Derivata prima e monotonia

        \[f'(x) = \frac{16-4x}{(x-2)^3}\]

    Il denominatore è negativo per x < 2 e positivo per x > 2, mentre il numeratore è negativo per x > 4 e positivo per x < 4; ne segue che:

        \[\quad\]

        \[\quad\]

        \[\quad\]

    Rendered by QuickLaTeX.com

        \[\quad\]

        \[\quad\]

    La funzione cresce su (2,4) e decresce su (-\infty, 2) \cup (4, +\infty). In particolare, il punto x = 4 risulta essere un massimo assoluto con f(4) = 1.

  12.  

  13. Derivata seconda e convessità

        \[f''(x) = \frac{8x-40}{(x-2)^4}\]

    Questa volta il denominatore è sempre positivo, perciò il segno dipende unicamente dal numeratore come segue:

        \[\quad\]

        \[\quad\]

        \[\quad\]

    Rendered by QuickLaTeX.com

        \[\quad\]

        \[\quad\]

    La funzione è convessa su (5, +\infty) e concava su (-\infty, 2) \cup (2,5). Punto di flesso in \left(5, \dfrac{8}{9}\right).

  14.  

  15. Grafico

        \[\quad\]

        \[\quad\]

        \[\quad\]

    Rendered by QuickLaTeX.com


 
 

Esercizio 14  (\bigstar\largewhitestar\largewhitestar\largewhitestar\largewhitestar). Sia

    \[ f(x) = \sqrt{x^3-3x}. \]

Tracciare il grafico della funzione dopo aver determinato: il dominio D, intersezione con gli assi, positività, continuità, asintoti, massimi e minimi, eventuali punti di non derivabilità, flessi, concavità e convessità. Una volta tracciato il grafico, esplicitare l’immagine di f.

Soluzione.

  1. Dominio Il dominio è determinato dalla condizione x^3-3x \geq 0. Il termine a sinistra si può scrivere come il prodotto x(x^2-3), perciò basta studiare il segno come segue:

        \[\quad\]

        \[\quad\]

        \[\quad\]

    Rendered by QuickLaTeX.com

        \[\quad\]

        \[\quad\]

    Il prodotto è positivo quando i segni dei due termini coincidono; in particolare, il dominio della funzione è dato da D = [-\sqrt 3, 0] \cup [ \sqrt 3, + \infty).

  2.  

  3. Simmetria La funzione non presenta simmetrie. Questo è immediato da verificare dato che, ad esempio, il dominio non è simmetrico.
  4.  

  5. Intersezioni con gli assi

        \[\quad\]

    • Asse x: La radice si annulla negli zeri del prodotto x(x^2-3), perciò interseca l’asse x nei punti O=(0,0), A=(\sqrt 3, 0) e B=(-\sqrt 3, 0).
    •  

    • Asse y: Si è già verificato sopra, l’intersezione avviene nell’origine O.
  6.  

  7. Positività La funzione è sempre non negativa sul suo dominio in quanto radice quadrata. Infatti, ricordiamo che la radice quadrata soddisfa

        \[ 	\sqrt{ h(x) } \ge 0 \qquad \text{per ogni $x$ nel dominio}. 	\]

  8.  

  9. Asintoti

        \[\quad\]

    • Asintoto orizzontale: Si vede che

          \[ 		\lim_{x \to + \infty} f(x) = + \infty, 		\]

      mentre per x \to - \infty la funzione non è definita; di conseguenza, non ammette asintoti orizzontali.

    •  

    • Asintoti verticali: Non esistono.
    •  

    • Asintoti obliqui: Non ci sono asintoti obliqui poiché è facile verificare che

          \[ 		\lim_{x \to + \infty} \frac{f(x)}{x} = + \infty 		\]

      in quanto f(x) è asintoticamente \sqrt{x^3} per x grande, quindi di ordine superiore rispetto ad x al denominatore.

  10.  

  11. Derivata prima e monotonia

        \[f'(x) = \frac{3(x^2-1)}{2\sqrt{x^3-3x}}\]

    Il denominatore è sempre positivo, quindi è sufficiente considerare il segno del numeratore. Si ha

        \[ 	x^2 - 1 > 0 \iff x \in (-\infty,-1) \cup (1,+\infty), 	\]

    perciò intersecando con il dominio della funzione si verifica immediatamente che:

        \[\quad\]

    • f'(x) > 0 per x \in (-\sqrt{3},-1) \cup (\sqrt{3},+\infty)
    •  

    • f'(x) < 0 per x \in (-1,0)

    La funzione cresce su (-\sqrt{3},-1) \cup (\sqrt{3},+\infty) e decresce su (-1,0). Dunque, in x=-1 c’è un massimo relativo con

        \[ 	f(-1) = \sqrt 2, 	\]

    mentre (0,0) e (\pm\sqrt{3},0) sono punti di minimo assoluto. Si noti che la funzione risulta non derivabile in x=0 e x=\pm\sqrt{3}.

  12.  

  13. Derivata seconda e convessità

        \[f''(x) = \frac{3(x^4-6x^2-3)}{4\sqrt{(x^3-3x)^3}}\]

    Come per la derivata prima, il denominatore non gioca alcun ruolo nello studio del segno. Per il numeratore, invece, prendiamo t=x^2 e troviamo

        \[ 	t^2 - 6t - 3 = 0 \iff t_{1,2} = \frac{6 \pm \sqrt{36+12}}{2} = 3 \pm 2 \sqrt 3. 	\]

    Dato che t = x^2 deve essere positivo, scartiamo la soluzione negativa. Si trova perciò che

        \[ 	x^4 - 6x^3 - 3 > 0 \iff x \in (-\infty, - \sqrt{3 + 2 \sqrt3}) \cup (\sqrt{3 + 2 \sqrt3}, + \infty). 	\]

    Intersecando con il dominio della funzione, deduciamo

        \[\quad\]

    • f''(x) > 0 per x \in (\sqrt{3+2\sqrt{3}},+\infty)
    •  

    • f''(x) < 0 per x \in (-\sqrt{3},0) \cup (\sqrt{3},\sqrt{3+2\sqrt{3}})

    La funzione è convessa su (\sqrt{3+2\sqrt{3}},+\infty) e concava su (-\sqrt{3},0) \cup (\sqrt{3},\sqrt{3+2\sqrt{3}}), perciò ammette un punto di flesso in

        \[ 	F = (\sqrt{3+2\sqrt{3}},3\sqrt{14+8\sqrt{3}}). 	\]

  14.  

  15. Grafico

    \[\quad\]

    \[\quad\]

    \[\quad\]

Rendered by QuickLaTeX.com


 
 

Esercizio 15  (\bigstar\bigstar\bigstar\largewhitestar\largewhitestar). Sia

    \[  f(x)=\frac{4|x-1|-x^2}{|x+1|}. \]

Tracciare il grafico della funzione dopo aver determinato: il dominio D, intersezione con gli assi, positività, continuità, asintoti, massimi e minimi, eventuali punti di non derivabilità, flessi, concavità e convessità. Una volta tracciato il grafico, esplicitare l’immagine di f.

Soluzione.

  1. Dominio La funzione risulta non essere definita solo nei punti in cui si annulla il denominatore. In particolare,

        \[ 	|x+1| \neq 0 \iff x \neq - 1,  	\]

    per cui il dominio è D=(-\infty,-1)\cup(-1,+\infty).

  2.  

  3. Simmetria La funzione non presenta simmetrie.
  4.  

  5. Intersezioni con gli assi

        \[\quad\]

    • Asse y: Si ha

          \[ 		f(0)=\frac{4 |-1|}{|1|} = 4, 		\]

      perciò A=(0,4) è il punto di intersezione con l’asse y.

    •  

    • Asse x: Basta verificare dove si annulla il numeratore. Sviluppando il modulo si ha

          \[ 		4 |x-1| - x^2 = \begin{cases} 			4x - 4 - x^2 & \text{se } x \ge 1, 			\\ -4x+4-x^2 & \text{se } x < 1. 		\end{cases} 		\]

      Nel primo caso, ovvero per x \ge 1, abbiamo

          \[ 		x^2 - 4x + 4 = 0 \iff (x-2)^2 = 0, 		\]

      che ha come unica soluzione x = 2 (accettabile, dato che rientra nell’intervallo (1,+\infty)). Nel secondo caso, si ha

          \[ 		x^2 + 4x - 4 = 0 \leadsto x_{1,2} = \frac{-4 \pm \sqrt{32}}{2} = - 2 \pm 2 \sqrt 2, 		\]

      ed entrambe soddisfano la condizione x < 1. Perciò otteniamo tre punti di intersezione con l’asse x, ovvero B=(2,0), C=(- 2 - 2 \sqrt2, 0) e D=(-2 + 2 \sqrt 2, 0).

  6.  

  7. Positività Il denominatore è sempre positivo perché valore assoluto, quindi ci concentriamo sul numeratore. In base ai conti fatti per l’intersezione con gli assi, deduciamo subito che:

        \[f(x)>0 \iff x\in(-2(1+\sqrt{2}),-1)\cup(-1,2(\sqrt{2}-1))\]

        \[f(x)<0 \iff x\in(-\infty,-2(1+\sqrt{2}))\cup(2(\sqrt{2}-1),+\infty)\]

  8.  

  9. Asintoti

        \[\quad\]

    • Asintoti orizzontali: Al numeratore il termine dominante è -x^2, perciò

          \[ 		\lim_{x \to \pm \infty} f(x) = - \lim_{x \to \pm \infty} \frac{x^2}{|x+1|} = - \infty, 		\]

      perciò non ci sono asintoti orizzontali.

    •  

    • Asintoti obliqui: Nel caso x \to + \infty il coefficiente angolare è

          \[ 		m_+ = \lim_{x \to + \infty} \frac{f(x)}{x} = \lim_{x \to + \infty} \frac{-x^2}{x|x+1|} = - \lim_{x \to + \infty} \frac{x}{x+1} = -1, 		\]

      da cui segue che

          \[ \begin{split} 		q_+ &= \lim_{x \to + \infty}  f(x) - m_+ x \\ & = \lim_{x \to + \infty} f(x) + x \\ &= \lim_{x \to + \infty}  \frac{4(x-1) - x^2 + x(x+1)}{x+1} \\ &= \lim_{x \to + \infty} \frac{4x + x}{x} \\ &= 5, \end{split} 		\]

      perciò y = - x +5 è asintoto obliquo per x \to + \infty. Nel caso x \to -\infty, invece, il coefficiente angolare è

          \[ 		m_- = \lim_{x \to - \infty} \frac{f(x)}{x} = \lim_{x \to - \infty} \frac{-x^2}{x|x+1|} = \lim_{x \to - \infty} \frac{x}{x+1} = 1, 		\]

      da cui segue che

          \[ \begin{split} 		q_- &= \lim_{x \to - \infty}  f(x) - m_- x \\ &= \lim_{x \to - \infty} f(x) - x \\ &= \lim_{x \to - \infty}  \frac{-4(x-1) - x^2 + x(x+1)}{-x-1} \\ &= \lim_{x \to + \infty} \frac{-3x}{-x} \\ &= 3, \end{split} 		\]

      perciò y = x +3 è asintoto obliquo per x \to - \infty.

    •  

    • Asintoto verticale: Verifichiamo i limiti destro e sinistro nel punto di discontinuità x=-1. Il denominatore è positivo da entrambi i lati, perciò:

          \[ 		\lim_{x \to -1^-} f(x) = \lim_{x \to -1^+} f(x) = \frac7{0^+} = + \infty, 		\]

      da cui segue che x=1 è un asintoto verticale.

  10.  

  11. Derivata prima e monotonia In questo caso, sviluppiamo i moduli per scrivere

        \[ 	 f(x) = \begin{cases} 		- \dfrac{4(1-x) - x^2}{x+1} & \text{se } x \le -1 		\\[9pt] \dfrac{4(1-x) - x^2}{x+1} & \text{se } -1 < x \le 1 		\\[9pt] \dfrac{4(x-1) - x^2}{x+1} & \text{se } x > 1. 	 \end{cases} 	\]

    La derivata prima si svolge in maniera simile in tutti e tre i casi:

        \[ 	f'(x) = \begin{cases} 			\dfrac{8 + 2 x + x^2}{(1 + x)^2} & \text{se } x < -1 			\\[9pt] - \dfrac{8 + 2 x + x^2}{(1 + x)^2} & \text{se } -1 < x < 1 			\\[9pt] - \dfrac{-8 + 2 x + x^2}{(1 + x)^2} & \text{se } x > 1. 	\end{cases} 	\]

    Osserviamo subito che nel caso x < -1 il segno è determinato dal numeratore. Si ha

        \[ 	x^2 + 2x + 8 = 0 \iff x_{1,2} = \frac{-2 \pm \sqrt{4 - 32}}{2}, 	\]

    ed essendo il discriminante negativo, il polinomio x^2+2x+8 è sempre positivo. In particolare, f'(x) è positivo in (-\infty,-1) e negativo in (-1,1). Per x>1, invece, si ha

        \[ 	x^2 + 2x - 8 = 0 \iff x_{1,2} = \frac{-2 \pm \sqrt{4 + 32}}{2} = -1 \pm 3, 	\]

    da cui si prende solo la soluzione positiva x_1=2 e si trova che la derivata è positiva in (1,2) e negativa in (2,+\infty). Ricapitolando, abbiamo:

        \[f'(x)>0 \iff x\in(-\infty,-1)\cup(1,2)\]

        \[f'(x)<0 \iff x\in(-1,1)\cup(2,+\infty)\]

    Dato che la funzione è crescente in (-\infty,-1)\cup(1,2) e decrescente in (-1,1)\cup(2,+\infty), il punto (2,0) risulta essere un massimo relativo, mentre (1,-1/2) un minimo relativo.

  12.  

  13. Non derivabilità e punti angolosi Verifichiamo il limite da destra e sinistra in x=1 della derivata definita a tratti. Si ha

        \[ 	\lim_{x \to 1^-} f'(x) = -\lim_{x \to 1}\frac{8 + 2 x + x^2}{(1 + x)^2} = -\frac{11}{4}, 	\]

    e, analogamente,

        \[ 	\lim_{x \to 1^+} f'(x) = -\lim_{x \to 1}\frac{-8 + 2 x + x^2}{(1 + x)^2} = -\frac{-5}{4} = \frac54, 	\]

    perciò x=1 risulta essere un punto angoloso.

  14.  

  15. Derivata seconda e convessità La derivata seconda si può calcolare a tratti derivando individualmente i tre pezzi della derivata prima. Un semplice conto mostra che:

        \[f''(x)>0 \iff x\in(-\infty,-1)\cup(-1,1)\]

        \[f''(x)<0 \iff x\in(1,+\infty)\]

    In particolare, la funzione è convessa su (-\infty,-1)\cup(-1,1) e concava su (1,+\infty).

  16.  

  17. Grafico

    \[\quad\]

    \[\quad\]

    \[\quad\]

Rendered by QuickLaTeX.com


 
 

Esercizio 16  (\bigstar\bigstar\largewhitestar\largewhitestar\largewhitestar). Sia

    \[  f(x)=\log\left(e+\frac{1}{x}\right). \]

Tracciare il grafico della funzione dopo aver determinato: il dominio D, intersezione con gli assi, positività, continuità, asintoti, massimi e minimi, eventuali punti di non derivabilità, flessi, concavità e convessità. Una volta tracciato il grafico, esplicitare l’immagine di f.

Soluzione.

  1. Dominio Il dominio è determinato ponendo il denominatore diverso da zero e l’argomento del logaritmo positivo. In altre parole, otteniamo il sistema:

        \[\left\{\begin{array}{l} 		x\neq 0\\ 		e+1/x>0 		\end{array}\right.\]

    Risolvendo, otteniamo:

        \[D=\left(-\infty,-1/e\right)\cup(0,+\infty)\]

  2.  

  3. Intersezioni con gli assi Per l’intersezione con l’asse delle x, cerchiamo le soluzioni dell’equazione

        \[ 		\log(e + 1/x) = 0 \iff e + 1/x = 1 \iff x = \frac{1}{1-e}. 		\]

    Dunque la funzione interseca l’asse delle x in A=(1/(1-e),0). Non ci sono intersezioni con l’asse y dato che x=0 non appartiene al dominio.

  4.  

  5. Positività La funzione è positiva quando l’argomento del logaritmo è maggiore di uno, ovvero in:

        \[(-\infty,1/(1-e))\cup(0,+\infty)\]

  6.  

  7. Asintoti

        \[\quad\]

    • Asintoto orizzontale: E’ facile vedere che

          \[ 			\lim_{x \to \pm \infty} \log(e+ 1/x) = \log e = 1, 			\]

      quindi y = 1 è un asintoto orizzontale per x \to \pm \infty.

    •  

    • Asintoti verticali: Consideriamo i due punti di discontinuità -1/e e 0 da sinistra e da destra rispettivamente. Nel primo caso, si ha

          \[ 			\lim_{x \to (-1/e)^-} \log(e + 1/x) = - \infty, 			\]

      mentre nel secondo caso

          \[ 			\lim_{x \to 0^+} \log(e + 1/x) = \lim_{x \to 0^+} \log(1/x) = + \infty. 			\]

      In particolare, x = -1/e è un asintoto verticale sinistro e x=0 un asintoto verticale destro.

  8.  

  9. Derivata prima e monotonia Si ha

        \[f'(x)=-\frac{1}{x(ex+1)}.\]

    La funzione è sempre decrescente sul suo dominio poiché f'(x)<0 per ogni x nel dominio. Infatti, il segno dipende unicamente dal denominatore:

        \[ 		x(ex + 1) > 0 \iff \begin{cases} x > 0 \\ x > - 1/e \end{cases} \text{ oppure } \begin{cases} 			x < 0 \\ x < -1/e 		\end{cases} 		\]

    E’ facile verificare che il primo sistema ha soluzione x>0 ed il secondo x < -1/e, che insieme corrispondono al dominio di f.

  10.  

  11. Derivata seconda e convessità Si ha

        \[f''(x)=\frac{2ex+1}{x^2(ex+1)^2}.\]

    In questo caso, il segno è determinato dal numeratore, ovvero

        \[ 		2ex + 1 > 0 \iff x > - \frac{1}{2e}, 		\]

    da cui segue che:

        \[\quad\]

    • f''(x)>0 per x>-1/(2e)
    •  

    • f''(x)<0 per x<-1/(2e)

    Intersecando con il dominio della funzione, deduciamo che f è convessa su (0,+\infty) e concava su (-\infty,-1/e). Non ci sono punti di flesso dato che -1/(2e) non appartiene al dominio.

  12.  

  13. Grafico

    \[\quad\]

    \[\quad\]

    \[\quad\]

Rendered by QuickLaTeX.com


 
 

Esercizio 17  (\bigstar\largewhitestar\largewhitestar\largewhitestar\largewhitestar). Sia

    \[ f(x)=\cos 2x-\sin x. \]

Tracciare il grafico della funzione dopo aver determinato: il dominio D, intersezione con gli assi, positività, continuità, asintoti, massimi e minimi, eventuali punti di non derivabilità, flessi, concavità e convessità. Una volta tracciato il grafico, esplicitare l’immagine di f.

Soluzione.

  1. Dominio Il dominio è \mathbb{R} dato che f è somma di funzioni trigonometriche. Il periodo si calcola facilmente osserando che

        \[ 		f(x+T)=f(x) \iff \cos(2x + T) = \cos2x \text{ e } \sin (x+T) = \sin x, 		\]

    dunque T = 2\pi. Per questo motivo, limitiamo lo studio di funzione all’intervallo [0,2\pi).

  2.  

  3. Asintoti Non ci sono punti di discontinuità (perciò sono assenti asintoti verticali). Dalla periodicità segue che non ci sono neanche asintoti orizzontali/obliqui.
  4.  

  5. Segno e intersezioni con gli assi Il segno della funzione si determina risolvendo la disequazione trigonometrica

        \[ 		\cos 2x - \sin x > 0. 		\]

    Per risolverla, si può sfruttare l’identità trigonometrica \cos 2x = 1 - 2 \sin^2 x. In tal caso, la disequazione si riscrive come segue:

        \[ 		1 - 2 \sin^2 x - \sin x > 0. 		\]

    Ponendo t = \sin x si risolve come una normale disequazione di secondo grado, e poi si torna alla variabile x. Un breve conto mostra che:

        \[f(x)>0 \iff x\in\left[0,\frac{\pi}{6}\right)\cup\left(\frac{5\pi}{6},2\pi\right)\]

        \[f(x)<0 \Leftrightarrow x\in\left(\frac{\pi}{6},\frac{5\pi}{6}\right)\]

    L’intersezione con l’asse delle y è nel punto A=(0,1), mentre le intersezioni con l’asse delle x si trovano come soluzioni dell’equazione sopra e sono B=\left(\dfrac{\pi}{6},0\right), C=\left(\dfrac{5\pi}{6},0\right) e D=\left(\dfrac{3\pi}{2},0\right).

  6.  

  7. Derivata prima e monotonia Ricordando che \sin 2x = 2 \sin x \cos x, abbiamo:

        \[f'(x)=-2\sin 2x-\cos x=-\cos x(4\sin x+1)\]

    La funzione cresce su (\pi/2,\pi+\alpha)\cup(3\pi/2,2\pi-\alpha) e decresce altrove, dove \alpha è l’angolo che soddisfa l’uguaglianza \sin\alpha=1/4. In particolare:

        \[\quad\]

    • I punti \left(\pi+\alpha,\dfrac{9}{8}\right), \left(2\pi-\alpha,\dfrac{9}{8}\right) sono massimi assoluti.
    •  

    • I punti \left(\dfrac{\pi}{2},-2\right) e \left(\dfrac{3\pi}{2},0\right) sono, rispettivamente, minimo assoluto e minimo relativo.
  8.  

  9. Derivata seconda e convessità Si ha

        \[f''(x)=8\sin^2 x+\sin x-4.\]

    Ponendo t = \sin x, risolviamo l’equazione di secondo grado associata ed osserviamo che la convessità di f cambia per valori di x che soddisfano:

        \[\sin x=\frac{-1\pm\sqrt{129}}{16}\]

  10.  

  11. Grafico La funzione si disegna tra [0,2\pi) e poi si estende per periodicità a tutto \mathbb R. Questo è visibile nel grafico, dove ogni periodo è identificato da linee verticali tratteggiate:

    \[\quad\]

    \[\quad\]

    \[\quad\]

Rendered by QuickLaTeX.com


 
 

Esercizio 18  (\bigstar\largewhitestar\largewhitestar\largewhitestar\largewhitestar). Sia

    \[ f(x)=\arcsin(1-x^2). \]

Tracciare il grafico della funzione dopo aver determinato: il dominio D, intersezione con gli assi, positività, continuità, asintoti, massimi e minimi, eventuali punti di non derivabilità, flessi, concavità e convessità. Una volta tracciato il grafico, esplicitare l’immagine di f.

Soluzione.

  1. Dominio e simmetrie La funzione \arcsin x è definita solo per x \in [-1,1], quindi in questo caso il dominio è determinato dalla condizione:

        \[ 		1 - x^2 \in [-1,1] \iff -1 \le 1-x^2 \le 1 \iff -2 \le -x^2 \le 0 \iff 0 \le x^2 \le 2, 		\]

    ovvero per x \in [-\sqrt 2, \sqrt 2]. In particolare,

        \[ 		D = [-\sqrt{2},\sqrt{2}]. 		\]

    Inoltre, è facile vedere che f è pari (f(-x)=f(x)). Di conseguenza, possiamo limitarci a studiare la funzione sul dominio D_+ = [0,\sqrt 2].

  2.  

  3. Asintoti La funzione non è definita per x \to + \infty, ed inoltre

        \[ 		\lim_{x \to + \sqrt 2} \arcsin (1-x^2) = \arcsin(-1) = - \frac\pi2, 		\]

    quindi non ci sono asintoti di alcun tipo.

  4.  

  5. Segno e intersezioni con gli assi La funzione è positiva quando l’argomento dell’arcoseno è positivo, perciò:

        \[ 		f(x) > 0 \iff 0 < x < 1. 		\]

    L’intersezione con l’asse delle y è A=(0,f(0))=(0,\pi/2), mentre le intersezioni con l’asse delle x si ottengono come soluzioni di

        \[ 		\arcsin(1-x^2) = 0, 		\]

    ovvero x = 1; in particolare, i punti di intersezione sono B=(1,0) e (per simmetria) C=(-1,0).

  6.  

  7. Derivata prima e monotonia Ricordiamo che

        \[ 		(\arcsin f)' = \frac{f^\prime}{\sqrt{1-f^2}}, 		\]

    da cui segue

        \[ 		f'(x)=-\frac{2x}{|x|\sqrt{2-x^2}}. 		\]

    Osserviamo che il denominatore della derivata si annulla nei punti x = 0, x = \sqrt{2} e x = -\sqrt{2}. Di conseguenza, la funzione potrebbe non essere derivabile in questi punti.

    In particolare, la funzione non è derivabile in x = 0, che risulta essere un punto angoloso. Infatti:

        \[ \lim_{x \to 0^-} \left(-\frac{2x}{|x|\sqrt{2-x^2}}\right) = \lim_{x \to 0^-} \frac{2}{\sqrt{2-x^2}} = \sqrt{2}, \]

    mentre

        \[ \lim_{x \to 0^+} \left(-\frac{2x}{|x|\sqrt{2-x^2}}\right) = \lim_{x \to 0^+} -\frac{2}{\sqrt{2-x^2}} = -\sqrt{2}. \]

    I limiti esistono, ma non coincidono, soddisfacendo la definizione di punto angoloso.

    Per quanto riguarda x = \sqrt{2} e x = -\sqrt{2}, abbiamo:

        \[ \lim_{x \to \sqrt{2}^-} \left(-\frac{2x}{|x|\sqrt{2-x^2}}\right) = -\infty, \]

    e

        \[ \lim_{x \to -\sqrt{2}^+} \left(-\frac{2x}{|x|\sqrt{2-x^2}}\right) = -\infty. \]

    Pertanto, in x = \pm \sqrt{2}, la funzione non è derivabile. In particolare, in questi punti la tangente è verticale. La funzione cresce su (-\sqrt{2},0) e decresce su (0,\sqrt{2}), perciò:

        \[\quad\]

    • Il punto (0,\pi/2) è un massimo assoluto.
    •  

    • I punti (\pm \sqrt 2, - \pi/2) sono minimi assoluti.
  8.  

  9. Grafico

    \[\quad\]

    \[\quad\]

    \[\quad\]

Rendered by QuickLaTeX.com


 
 

Esercizio 19  (\bigstar\largewhitestar\largewhitestar\largewhitestar\largewhitestar). Sia

    \[ f(x)=\arctan\frac{x+1}{x-1}. \]

Tracciare il grafico della funzione dopo aver determinato: il dominio D, intersezione con gli assi, positività, continuità, asintoti, massimi e minimi, eventuali punti di non derivabilità, flessi, concavità e convessità. Una volta tracciato il grafico, esplicitare l’immagine di f.

Soluzione.

  1. Dominio La funzione \arctan x è definita per ogni x, perciò è sufficiente porre il denominatore dell’argomento diverso da zero:

        \[  		D = (-\infty,1)\cup(1,+\infty). 		\]

  2.  

  3. Segno e intersezioni con gli assi La funzione è positiva quando l’argomento dell’arcotangente è positivo, perciò:

        \[ 		\frac{x+1}{x-1} > 0 \iff \begin{cases} x > -1 \\ x > 1 \end{cases} \text{ oppure } \begin{cases} x < - 1 \\ x < 1 \end{cases} 		\]

    e, risolvendo i due sistemi, la loro unione corrisponde a:

        \[ 		f(x) > 0 \iff x \in (-\infty,-1) \cup (1,+ \infty). 		\]

    L’intersezione con l’asse delle y avviene nel punto A=(0,f(0))=(0, - \pi/4), mentre con l’asse delle x basta osservare che l’arcotangente si annulla solo quando l’argomento è zero, perciò in B=(-1,0).

  4.  

  5. Asintoti

        \[\quad\]

    • Asintoto orizzontale: E’ facile vedere che

          \[ 			\lim_{x \to \pm \infty} \arctan \frac{x+1}{x-1} = \arctan 1 = \frac\pi4, 			\]

      perciò y = \pi/4 è un asintoto orizzontale per x \to \pm \infty.

    •  

    • Asintoto verticale: Nel punto di discontinuità x=1 abbiamo

          \[ 			\lim_{x \to 1^-} \arctan\dfrac{x+1}{x-1} = - \frac\pi2, 			\]

      mentre

          \[ 			\lim_{x \to 1^-} \arctan\dfrac{x+1}{x-1} = \frac\pi2. 			\]

      In particolare, x = 1 non è un asintoto verticale ma una discontinuità con salto.

  6.  

  7. Derivata prima e monotonia Ricordando che

        \[ 		(\arctan x)' = \frac{1}{x^2+1}, 		\]

    deduciamo immediatamente che:

        \[f'(x)=-\frac{1}{x^2+1}.\]

    La derivata è sempre negativa, perciò f decresce su tutto il dominio.

  8.  

  9. Derivata seconda e convessità Si ha

        \[f''(x)=\frac{2x}{(x^2+1)^2}.\]

    Il segno dipende dal numeratore, ovvero da x, perciò la funzione è convessa su (0,1)\cup(1,+\infty), concava su (-\infty,0). In particolare, c’è un punto di flesso in (0,-\pi/4)

  10.  

  11. Grafico

    \[\quad\]

    \[\quad\]

    \[\quad\]

Rendered by QuickLaTeX.com


 
 

Esercizio 20  (\bigstar\bigstar\largewhitestar\largewhitestar\largewhitestar). Studiare le seguenti funzioni e tracciarne un grafico qualitativo:

    \[\quad\]

  1. f(x)=\dfrac{x^3-x}{x^2-4}
  2.  

  3. f(x)=\dfrac{\log x}{x}
  4.  

  5. f(x)=2x+\sqrt{x^2-1}
  6.  

  7. f(x)=\dfrac{3x+1}{x+1}-2\arctan x
  8.  

  9. f(x)=(x^2-4)e^{-|x|}
  10.  

  11. f(x)=x\cdot\dfrac{2\log x-3}{\log x-2}

Soluzione punto 1.

Consideriamo la funzione

    \[ 		f(x) = \frac{x^3-x}{x^2-4}. 		\]

Questa è definita quando il denominatore non si annulla, ovvero x^2-4\neq 0. Nello specifico, ne segue che il dominio è dato da

    \[D=(-\infty,-2)\cup(-2,2)\cup(2,+\infty).\]

Inoltre

    \[f(-x)=\frac{(-x)^3-(-x)}{(-x)^2-4}=\frac{-x^3+x}{x^2-4}=-\frac{x^3-x}{x^2-4}=-f(x),\]

per cui la funzione è dispari e ciò permette di studiarla nel solo dominio D_+=[0,2)\cup(2,+\infty) e di ottenere il suo grafico su D\setminus D_+ per simmetria rispetto all’origine degli assi coordinati.

Per x=0 si ha f(0)=0, mentre f(x)=0 se e solo se

    \[x^3-x=0\iff x(x^2-1)=0\iff x=0,\ x=\pm 1,\]

e quindi la funzione interseca gli assi nei punti

    \[O=(0,0),\quad A=(1,0),\quad B=(-1,0).\]

Risolvendo f(x)>0 nella parte positiva del dominio (D_+) si trova che

    \[f(x)>0\iff x\in (0,1)\cup(2,+\infty).\]

Inoltre, dato che il numeratore è un polinomio di ordine maggiore rispetto al denominatore, non ci sono asintoti orizzontali e vale:

    \[ 		\lim_{x \to \pm \infty} f(x) = \pm \infty. 		\]

Per quanto riguarda gli asintoti verticali, invece, verifichiamo nel punto di discontinuità, ovvero x=2 (in quanto x=-2 si ottiene per simmetria). Si ha

    \[ 		\lim_{x\to 2^{\pm}} f(x)=\pm\infty, 		\]

da cui segue che le rette x=\pm 2 sono asintoti verticali per f(x). Dato che il grado del numeratore è maggiore del denominatore di uno, ci sono asintoti obliqui:

    \[m=\lim_{x\to + \infty}\frac{f(x)}{x}=\lim_{x\to\infty}\frac{x^3}{x\cdot x^2}=1,\]

    \[q=\lim_{x\to\infty}\left[f(x)-x\right]=\lim_{x\to\infty}\frac{x^3-x-x^3+4x}{x^2-4}= 		\lim_{x\to\infty}\frac{3x}{x^2}=0,\]

segue che la retta y=x è un asintoto obliquo sia destro che (per simmetria) sinistro. Per la derivata si può utilizzare la formula per la derivata di un rapporto per ottenere:

    \[f'(x)=\frac{(3x^2-1)(x^2-4)-(x^3-x)(2x)}{(x^2-4)^2}=\frac{x^4-11x^2+4}{(x^2-4)^2},\]

Il denominatore non gioca alcun ruolo nel determinare il segno di f', perciò è sufficiente risolvere la disequazione:

    \[x^4-11x^2+4 > 0.\]

Come al solito, ponendo t=x^2 si ottiene t^2-11t+4 > 0, che ha come soluzioni

    \[t < \frac{11-\sqrt{105}}{2}\quad \text{e} \quad t > \frac{11+\sqrt{105}}{2},\]

per cui ci riconduciamo alle disequazioni:

    \[x^2 < \frac{11-\sqrt{105}}{2}\quad \text{e} \quad x^2 > \frac{11+\sqrt{105}}{2}.\]

Si trova

    \[-\sqrt{\frac{11-\sqrt{105}}{2}}\leq x\leq+\sqrt{\frac{11-\sqrt{105}}{2}}\]

con la prima, e

    \[x\leq-\sqrt{\frac{11+\sqrt{105}}{2}},\quad x\geq+\sqrt{\frac{11+\sqrt{105}}{2}}\]

con la seconda. Poiché 11^2-105=16=4^2, si ha

    \[\sqrt{\frac{11-\sqrt{105}}{2}}=\frac{1}{\sqrt{2}}\left(\sqrt{11+4}{2}-\sqrt{11-4}{2}\right)=\frac{\sqrt{15}-\sqrt{7}}{2},\]

    \[\sqrt{\frac{11+\sqrt{105}}{2}}=\frac{1}{\sqrt{2}}\left(\sqrt{11+4}{2}+\sqrt{11-4}{2}\right)=\frac{\sqrt{15}+\sqrt{7}}{2},\]

da cui

    \[-\frac{\sqrt{15}-\sqrt{7}}{2}\leq x\leq\frac{\sqrt{15}-\sqrt{7}}{2},\]

    \[x\leq-\frac{\sqrt{15}+\sqrt{7}}{2},\quad x\geq\frac{\sqrt{15}+\sqrt{7}}{2},\]

per cui f'(x)\geq 0 se e solo se

    \[x\in\left(-\infty,-\frac{\sqrt{15}+\sqrt{7}}{2}\right]\cup\left[-\frac{\sqrt{15}-\sqrt{7}}{2},\frac{\sqrt{15}-\sqrt{7}}{2}\right] 		\cup\left[\frac{\sqrt{15}+\sqrt{7}}{2},+\infty\right),\]

e su tali intervalli la funzione risulta crescente. Inoltre f risulta avere dei massimi relativi nei punti

    \[M_1=\left(-\frac{\sqrt{15}+\sqrt{7}}{2},-\frac{7\sqrt{7}+5\sqrt{15}}{16}\right), \quad M_2=\left(\frac{\sqrt{15}-\sqrt{7}}{2},\frac{-7\sqrt{7}+5\sqrt{15}}{16}\right),\]

e dei minimi relativi nei punti

    \[m_1=\left(-\frac{\sqrt{15}-\sqrt{7}}{2},-\frac{-7\sqrt{7}+5\sqrt{15}}{16}\right), \quad m_2=\left(\frac{\sqrt{15}+\sqrt{7}}{2},\frac{7\sqrt{7}+5\sqrt{15}}{16}\right).\]

Per la derivata seconda si ha poi

    \[ \begin{aligned}  			f''(x) & =\frac{(4x^3-22x)(x^2-4)^2-(x^4-11x^2+4)2(x^2-4)(2x)}{(x^2-4)^4} 		\\ & =\frac{6x^3+72x}{(x^2-4)^3}=\frac{6x(x^2+12)}{(x^2-4)^3}. 		\end{aligned}\]

La disequazione f''(x)\geq 0 ha soluzione in D_+, dove il numeratore è sempre positivo, determinata unicamente dal denominatore:

    \[x \in (2,+\infty),\]

dove f risulta essere convessa. Inoltre f ammette un punto di flesso in O=(0,0) la cui tangente risulta la retta y=x/4. Il grafico della funzione è riportato in figura:

    \[\quad\]

    \[\quad\]

    \[\quad\]

Rendered by QuickLaTeX.com

Soluzione punto 2.

La funzione

    \[ 	f(x)= \frac{\log x}{x} 	\]

è definita per x>0, per cui D=(0,+\infty). Non presenta dunque simmetrie e si ha

    \[ 	f(x)=0 \iff \log x=0, 	\]

ovvero per x=1. Quindi f interseca l’asse delle x nel punto A=(1,0).

Il denominatore è sempre positivo nel dominio D, perciò f(x)>0 è equivalente a \log x > 0, per cui f risulta essere positiva su (1,+\infty). Facendo il limite in 0 da destra otteniamo:

    \[ 	\lim_{x\rightarrow 0^+}f(x) = - \infty, 	\]

mentre il limite a +\infty è dato da

    \[ 	\lim_{x \to + \infty} \frac{ \log x}{x} = 0. 	\]

In particolare, la retta x=0 è un asintoto verticale (da destra) mentre la retta y=0 un asintoto orizzontale per x\to + \infty. Abbiamo poi

    \[f'(x)=\frac{\dfrac{1}{x}\cdot x-\log x}{x^2}=\frac{1-\log x}{x^2},\]

per cui f'(x) > 0 dipende unicamente dal segno del numeratore, ovvero:

    \[1-\log x > 0 \iff \log x < 1\iff x < e.\]

Intersecando con il dominio di f, deduciamo che questa è crescente su (0,e), decrescente su (e,+\infty), e presenta un massimo (assoluto) nel punto M=(e,1/e).

La derivata seconda è data da

    \[f''(x)=\frac{-\dfrac{1}{x}\cdot x^2-(1-\log x)(2x)}{x^4}=\frac{2\log x-3}{x^3},\]

ed essendo il denominatore sempre positivo sul dominio, la disequazione f''(x) > 0 è equivalente a

    \[2\log x-3 > 0\iff \log x>\frac{3}{2}\iff x> e^{3/2}=\sqrt{e^3}.\]

La funzione è quindi convessa su (\sqrt{e^3},+\infty), concava su (0,\sqrt{e^3}), e presenta un flesso nel punto F=(\sqrt{e^3},3/(2\sqrt{e^3})). Il grafico è riportato in figura.

    \[\quad\]

    \[\quad\]

    \[\quad\]

Rendered by QuickLaTeX.com

Soluzione punto 3.

Il dominio della funzione

    \[ 	f(x) = 2x + \sqrt{x^2-1} 	\]

si ottiene imponendo la condizione x^2-1\geq 0, per cui

    \[ 	D=(-\infty,-1]\cup[1,+\infty). 	\]

La funzione non ha simmetrie in quanto somma di una funzione pari e di una dispari. Inoltre, non interseca l’asse delle ordinate (x=0 non appartiene al dominio), mentre

    \[ 	f(x)=0 \iff 2x+\sqrt{x^2-1}=0\iff x^2-1=4x^2\iff 3x^2+1=0 	\]

che non ha soluzioni. Quindi non ci sono intersezioni neanche con l’asse delle ascisse. Risolvendo f(x)>0 ci si riconduce alla coppia di sistemi:

    \[ \begin{cases} 		-2x < 0 \\ x^2 -1 \ge 0 	\end{cases} \quad \text{e} \qquad \begin{cases} 	-2x \ge 0 \\ x^2 - 1 > 4x^2 	\end{cases} 	\]

Il primo ha soluzione x\geq 1, mentre il secondo non ammette alcuna soluzione. La funzione risulta allora positiva su [1,+\infty).

La funzione non ammette asintoti orizzontali dato che il limite a \pm \infty è dato da

    \[ 	\lim_{x\to \pm \infty}f(x)= \lim_{x\to \pm \infty}\frac{4x^2-x^2+1}{2x-\sqrt{x^2-1}}= \pm \infty,\]

perciò verifichiamo eventuali asintoti obliqui. Per x \to - \infty, si trova il coefficiente angolare:

    \[m_-=\lim_{x\to-\infty}\frac{f(x)}{x}=\lim_{x\to-\infty}\frac{3x^2+1}{2x^2-x\sqrt{x^2-1}}= 	\lim_{x\to-\infty}\frac{\displaystyle x^2\left(3+\frac{1}{x^2}\right)}{\displaystyle x^2\left(2+\sqrt{1-\frac{1}{x^2}}\right)}=1,\]

per cui il termine noto è:

    \[q_-=\lim_{x\to-\infty}\left[f(x)- m_- x\right]=\lim_{x\to-\infty}\left[x+\sqrt{x^2-1}\right]= 	\lim_{x\to-\infty}\frac{x^2-x^2+1}{x-\sqrt{x^2-1}}=0.\]

Dunque, la retta y=x è un asintoto obliquo per x \to - \infty. Analogamente, calcoliamo il coefficiente angolare a destra:

    \[m_+=\lim_{x\to+\infty}\frac{f(x)}{x}=\lim_{x\to+\infty}\left[2+\sqrt{1-\frac{1}{x^2}}\right]=3.\]

Il termine noto risulta essere

    \[q_+=\lim_{x\to+\infty}\left[f(x)-m_+ x\right]=\lim_{x\to+\infty}\left[\sqrt{x^2-1}-x\right]= 	\lim_{x\rightarrow-\infty}\frac{x^2-1-x^2}{\sqrt{x^2-1}+x}=0,\]

pertanto la retta y=3x è un asintoto obliquo per x\to +\infty. Infine, si verifica facilmente che

    \[\lim_{x\rightarrow-1^-}f(x)=f(-1)=-2,\qquad \lim_{x\rightarrow 1+}f(x)=f(1)=2,\]

perciò la funzione non ammette asintoti verticali nei punti di discontinuità. La derivata prima è

    \[f'(x)=2+\frac{2x}{2\sqrt{x^2-1}}=2+\frac{x}{\sqrt{x^2-1}}=\frac{2\sqrt{x^2-1}+x}{\sqrt{x^2-1}}.\]

Il denominatore è sempre positivo sul dominio (escludendo i punti x=\pm 1, dove si annulla), per cui la disequazione f'(x)> 0 è del tutto equivalente a

    \[2\sqrt{x^2-1}+x> 0\iff \begin{cases}  	-x<0\\ x^2-1> 0 \end{cases}\qquad \text{oppure} \qquad 	 \begin{cases} 	-x> 0\\ 4x^2-4> x^2 \end{cases}\]

Il primo sistema ha come soluzione x> 1, il secondo x<-2/\sqrt{3}, per cui la funzione risulta crescente su

    \[\left(-\infty,-\frac{2\sqrt{3}}{3}\right)\cup(1,+\infty)\]

e decrescente su

    \[\left(-\frac{2\sqrt{3}}{3},-1\right).\]

In particolare, la funzione ammette un massimo relativo nel punto

    \[M = \left(-\frac{2\sqrt{3}}{3},-\sqrt{3}\right),\]

ed ammette due minimi relativi nei punti

    \[A = (-1,-2) \quad \text{e} \quad B = (1,2).\]

I limiti a \pm 1 (da destra e sinistra rispettivamente) danno

    \[\lim_{x\to1^+}f'(x)=+\infty,\qquad \lim_{x\to -1^-} f'(x) = -\infty,\]

perciò in tali punti la funzione ha tangente verticale (x=1 e x-=1 rispettivamente). Infine per la derivata seconda si ha

    \[f''(x)=\frac{\sqrt{x^2-1}-x\dfrac{2x}{2\sqrt{x^2-1}}}{x^2-1}=\frac{x^2-1-x^2}{\sqrt{(x^2-1)^3}}=-\frac{1}{\sqrt{(x^2-1)^3}},\]

la quale risulta sempre negativa sul dominio, perciò f è concava. Il grafico è riportato nella figura che segue:

    \[\quad\]

    \[\quad\]

    \[\quad\]

Rendered by QuickLaTeX.com

Soluzione punto 4.

La funzione

    \[ 	f(x) = \frac{3x+1}{x+1} - 2 \arctan x 	\]

è definita per ogni valore reale ad esclusione di x=-1, in cui il denominatore si annulla. Di conseguenza:

    \[D=(-\infty,-1)\cup(-1,+\infty).\]

Si ha f(0)=1, per cui interseca l’asse delle ordinate nel punto C=(0,1). Per determinare le soluzioni di f(x)=0 osserviamo che questa è equivalente a scrivere

    \[\frac{3x+1}{x+1}=2\arctan x.\]

D’altra parte, il termine di sinistra si può ridurre ulteriormente facendo la divisione tra polinomi

    \[\frac{3x+1}{x+1}=\frac{3x+3-2}{x+1}=3-\frac{2}{x+1}.\]

L’equazione si può ora risolvere valutando il grafico e facendo delle considerazioni:

    \[\quad\]

    \[\quad\]

    \[\quad\]

Rendered by QuickLaTeX.com

    \[\quad\]

    \[\quad\]

La funzione y_1=2\arctan x è limitata e, nello specifico, prende valori nell’intervallo (-\pi,\pi). La funzione y_2=3-\dfrac{2}{x+1}, invece, coincide con l’iperbole equilatera y=2/x traslata in modo da avere come asintoti le rette y=3,\ x=-1. Ne segue che

    \[ 	 y_1(x)>y_2(x) \quad \text{per } x \to + \infty 	 \]

e, dato che y_1(0)=0 < 3 = y_2(0), deve esistere \beta>0 in cui tali curve si incontrano (teorema di esistenza degli zeri). Analogamente, dato che

    \[ 	 y_1(-1)=2\arctan(-1)=-\pi/2 \text{ e } y_2\to-\infty \quad \text{per } x\to-1, 	 \]

ne segue che esiste -1<\alpha<0 per cui le funzioni si intersecano. Ne segue che la funzione f(x) interseca l’asse delle ascisse nei punti A=(\alpha,0) e B=(\beta,0), risulta positiva su

    \[ 	(-\infty,-1)\cup(\alpha,\beta) 	\]

e negativa su

    \[ 	(-1,\alpha)\cup(\beta,+\infty). 	\]

Si noti che è possibile stimare il valore di \alpha e \beta. In particolare,

    \[ 	\alpha \approx -0.488291298846247 \qquad \text{e} \qquad \beta \approx 3.02569706166677. 	\]

I limiti a \pm \infty si possono calcolare facilmente sfruttando il fatto che l’arcotangente è limitata e il rapporto tra polinomi ha lo stesso grado a numeratore e denominatore:

    \[\lim_{x\to\pm\infty}f(x)=\lim_{x\to\pm\infty}\left[\frac{3x}{x}-2\arctan x\right]=3-2\left(\pm\frac{\pi}{2}\right)=3\mp\pi.\]

Dunque, la funzione ammette le rette y=3-\pi ed y=3+\pi come asintoti orizzontali a destra e sinistra rispettivamente. Nel punto di discontinuità x=-1 possiamo calcolare i limiti

    \[\lim_{x\rightarrow-1^{\pm}}f(x)=\mp\infty+2\left(-\frac{\pi}{4}\right)=\mp\infty,\]

per cui la funzione ha in x=-1 un asintoto verticale. La derivata prima è data da

    \[f'(x)=\frac{3(x+1)-(3x+1)}{(x+1)^2}-\frac{2}{x^2+1}=\frac{2}{(x+1)^2}-\frac{2}{x^2+1}=\frac{-4x}{(x^2+1)(x+1)^2},\]

perciò il segno della derivata dipende unicamente dal numeratore. In particolare, la funzione f risulta crescente su (-\infty,-1)\cup(-1,0), decrescente su (0,+\infty) e ammette un massimo relativo in C=(0,1). La derivata seconda è data da

    \[\begin{aligned} 	f''(x) & =\frac{-4(x^2+1)(x+1)^2+4x\left[2x(x+1)^2+(x^2+1)2(x+1)\right]}{(x^2+1)^2(x+1)^4} 	\\ & =\frac{-4x^3-4x^2-4x-4+16x^3+8x^2+8x}{(x^2+1)^2(x+1)^3} 	 \\ & =\frac{4(3x^3+x^2+x-1)}{(x^2+1)^2(x+1)^3}. 	\end{aligned}\]

Il denominatore di tale derivata risulta positivo per x\in(-1,+\infty). Per il numeratore, invece, si osservi che

    \[ 	3x^3+x^2+x-1 = 0 	\]

ha un’unica soluzione, che si può ricavare con qualsiasi metodo standard di risoluzione per equazioni del terzo ordine:

    \[ 	\gamma = \frac19 \left( -1 - \frac{4 \cdot 2^{2/3}}{\sqrt[3]{67 + 9 \sqrt{57}}} + \sqrt[3]{2 (67 + 9 \sqrt{57})} \right). 	\]

Ne segue che f''(x)>0 per x\in(-\infty,-1)\cup(\gamma,+\infty), dove la funzione è convessa, f''(x)<0 per x\in(-1,\gamma), dove la funzione è concava, ed f ammette un flesso nel punto F=(\gamma,f(\gamma)). Il grafico della funzione è riportato nella figura seguente:

    \[\quad\]

    \[\quad\]

    \[\quad\]

Rendered by QuickLaTeX.com

Soluzione punto 5.

Il dominio della funzione

    \[ 	f(x)=(x^2-4) e^{-|x|} 	\]

coincide con tutto l’asse reale. Inoltre, la funzione soddisfa

    \[ 	f(-x)=((-x)^2-4)e^{-|-x|}=(x^2-4)e^{-|x|}=f(x), 	\]

ovvero è pari. Inoltre f(0)=-4, mentre f(x)=0 se e solo se

    \[x^2-4=0\implies x=\pm 2.\]

La funzione interseca pertanto gli assi nei punti A=(0,-4),\ B=(-2,0) e C=(2,0). Dato che l’esponenziale è sempre positiva, f(x)>0 quando x^2-4>0, ovvero

    \[ 	f(x) > 0 \iff x\in(-\infty,-2)\cup(2,+\infty). 	\]

Per simmetria, possiamo limitarci al limite per x \to + \infty:

    \[\lim_{x\to+\infty}f(x)=[+\infty]\cdot [0],\]

dato che (x^4-4) tende ad infinito ed e^{-x} tende a zero. Questa è una forma indeterminata, perciò applichiamo de l’Hôpital due volte ed otteniamo:

    \[\lim_{x\to+\infty}f(x)=\lim_{x\to+\infty}\frac{x^2-4}{e^x}=\lim_{x\to+\infty}\frac{2}{e^x}=0,\]

per cui la retta y=0 è un asintoto orizzontale per x \to \pm \infty. Non ci sono punti di discontinuità, dunque non esistono asintoti verticali.

Passando al calcolo della derivata prima, per simmetria possiamo limitarci all’intervallo (0,+\infty) e sviluppare il modulo all’esponente (|x|=x); si ha:

    \[f'(x)=-e^{-x}(x^2-4)+2x e^{-x}=e^{-x}(4+2x-x^2),\]

per cui f'(x)\geq 0 su (0,+\infty) equivale a

    \[4+2x-x^2\geq 0\iff x^2-2x-4\leq 0\iff 1-\sqrt{5}\leq x\leq 1+\sqrt{5}.\]

In particolare, la derivata prima è positiva su (0,1+\sqrt{5}]. La funzione è crescente in (0,1+\sqrt{5}) e decrescente in (1+\sqrt{5},+\infty) (per simmetria, cresce su (-\infty,-1-\sqrt{5}) e decresce su (-1-\sqrt{5},)) ed ammette due massimi (assoluti) nei punti

    \[M_1=\left(-(1+\sqrt{5}),(5+2\sqrt{5})e^{-(1+\sqrt{5})}\right),\qquad M_2=\left(1+\sqrt{5},(5+2\sqrt{5})e^{-(1+\sqrt{5})}\right),\]

ed un minimo assoluto nel punto

    \[A=(0,-4).\]

Inoltre, dato che il modulo |x| non è differenziabile in x=0, per verificare la differenziabilità facciamo i limiti destro e sinistro del rapporto incrementale:

    \[ 	\lim_{x\to 0^{\pm}}\frac{f(x)-f(0)}{x} =\left\{\begin{array}{l} 	\displaystyle\lim_{x\to 0^{+}}\frac{(x^2-4)e^{-x}+4}{x}=-4 \lim_{x\to 0^{+}}\frac{e^{-x}-1}{x}=4\\ \\ 	\displaystyle\lim_{x\to 0^{-}}\frac{(x^2-4)e^{x}+4}{x}=-4\lim_{x\to 0^{-}}\frac{e^{x}-1}{x}=-4 	\end{array}\right.\]

I limiti esistono e sono finiti, ma non coincidono: dunque x=0 è un punto angoloso per f. La derivata seconda per x>0 è data da

    \[ 	f''(x)=-e^{-x}(4+2x-x^2)+e^{-x}(2-2x)=e^{-x}(x^2-4x-2), 	\]

per cui risolvendo f''(x) > 0 con la condizione x > 0 si ricava

    \[ 	x^2-4x-2 > 0\iff x< 2-\sqrt{6} \text{ oppure } x> 2+\sqrt{6}\iff x > 2+\sqrt{6}. 	\]

La funzione è pertanto convessa su (2+\sqrt{6},+\infty) e per simmetria su (-\infty,-2-\sqrt{6}), concava su (-2-\sqrt{6},2+\sqrt{6}), ed ammette flessi nei punti

    \[ 	F_1=\left(-(2+\sqrt{6}),(9+8\sqrt{6})e^{-(2+\sqrt{6})}\right),\qquad F_2=\left(2+\sqrt{6},(9+8\sqrt{6})e^{-(2+\sqrt{6})}\right), 	\]

in cui le tangenti hanno coefficiente angolare \pm 2(1+\sqrt{6}) rispettivamente. Il grafico della funzione è riportato nella figura seguente:

    \[\quad\]

    \[\quad\]

    \[\quad\]

Rendered by QuickLaTeX.com

Soluzione punto 6.

La funzione

    \[ 	f(x) = x \cdot \frac{2 \log x - 3}{\log x - 2} 	\]

è definita per x > 0 (per il logaritmo) e denominatore diverso da zero, ovvero

    \[ 	\log x-2\neq 0 \iff \log x\neq 2 \iff x\neq e^2. 	\]

In particolare, il dominio è

    \[D=(0,e^2)\cup(e^2,+\infty).\]

La funzione non è simmetrica e interseca l’asse delle ascisse quando

    \[ 	2\log x-3=0\implies x=e^{3/2}, 	\]

ovvero nel punto A=(e^{3/2},0). Inoltre, la funzione è positiva nell’unione di intervalli

    \[x\in(0,e^{3/2})\cup(e^2,+\infty).\]

Il limite a + \infty si calcola osservando che il secondo fattore tende a 2:

    \[ 	\lim_{x\to+\infty}f(x)=\lim_{x\to+\infty}x\cdot \underbrace{ \frac{2\log x}{\log x}}_{=2} = +\infty. 	\]

Il coefficiente angolare di un eventuale asintoto obliquo a destra è dunque

    \[ 	m=\lim_{x\to+\infty}\frac{f(x)}{x}=\lim_{x\to+\infty}\frac{2\log x}{\log x}=2, 	\]

mentre il termine noto

    \[ 	q=\lim_{x\to+\infty}\left[f(x)- m x\right]=\lim_{x\to+\infty}\frac{2x\log x-3x-2x\log x+4x}{\log x-2}= 	\lim_{x\to+\infty}\frac{x}{\log x-2}. 	\]

A questo punto si applica la regola di de l’Hôpital, ottenendo:

    \[q =\lim_{x\to+\infty}\frac{1}{1/x}=+\infty,\]

ovvero non c’è alcun asintoto obliquo. Il limite all’altro estremo del dominio di definizione è

    \[\lim_{x\to 0^+}f(x)=[0]\cdot\frac{[\infty]}{[\infty]},\]

perciò introduciamo la variabile ausiliaria t=1/x (in modo che t\to+\infty quando x\to 0^+) e utilizziamo la proprietà del logaritmo

    \[ 	\log \frac1t = - \log t 	\]

per arrivare a

    \[\lim_{x\to 0^+}f(x)=\lim_{t\to+\infty}\frac{1}{t}\cdot\frac{-2\log t-3}{-\log t-2}= 	\lim_{t\to +\infty}\frac{1}{t}\cdot\frac{2\log t}{\log t}=0.\]

In particolare, in x=0 c’é una discontinuità eliminabile ponendo f(0)=0. Infine, nel punto di discontinuità abbiamo

    \[\lim_{x\rightarrow (e^2)^{\pm}} f(x)=\pm\infty,\]

perciò la retta x=e^2 è un asintoto veritcale. Per la derivata prima abbiamo

    \[\begin{aligned} 	f'(x)&=\frac{\displaystyle\left[2\log x-3+x\cdot\frac{2}{x}\right](\log x-2)-x(2\log x-3)\frac{1}{x}}{(\log x-2)^2} 	\\ & = \frac{(2\log x-1)(\log x-2)-2\log x+3}{(\log x-2)^2} 	\\ & = \frac{2\log^2 x-7\log x+5}{(\log x-2)^2}. 	\end{aligned}\]

La disequazione f'(x)\geq 0, essendo il denominatore sempre positivo sul dominio, equivale a richiederla per il numeratore:

    \[2\log^2 x-7\log x+5\geq 0\]

sotto la condizione x>0 ed x \neq e^2. Ponendo t=\log x si ottiene una disequazione di secondo grado

    \[ 	2t^2-7t+5\geq 0\iff t\leq 1 \text{ oppure } t\geq\frac{5}{2} 	\]

e tornando alla variabile x corrispondono a:

    \[\quad\]

  • \log x \le 1, che ha soluzione x \le e;
  •  

  • \log x \ge 5/2, che ha soluzione x \ge e^{5/2}.

Riassumendo, la funzione cresce su (0,e)\cup(e^{5/2},+\infty) e decresce su (e,e^2)\cup(e^2,e^{5/2}). In particolare, f ha un massimo relativo in M=(e,e) ed un minimo relativo in m=(e^{5/2},4e^{5/2}). Inoltre, si ha

    \[ 	f'(0)=\lim_{x\to 0^+}f'(x)=\lim_{x\to 0^+}\frac{2\log^2 x-7\log x+5}{(\log x-2)^2}, 	\]

ed utilizzando ancora una volta la variabile ausiliaria t=\log x (per cui x\to 0^+ corrisponde a t\rightarrow -\infty) si trova

    \[ 	f'(0)=\lim_{t\to-\infty}\frac{2t^2-7t+5}{(t-2)^2}=\lim_{t\to-\infty}\frac{2t^2}{t^2}=2. 	\]

La funzione risulta perciò essere tangente alla retta y=2x nell’origine. Infine, la derivata seconda è data da

    \[ \begin{aligned} 	f''(x) & =\frac{\displaystyle\left(\frac{4\log x}{x}-\frac{7}{x}\right)(\log x-2)^2-(2\log^2 x-7\log x+5)\cdot 2(\log x-2)\cdot\frac{1}{x}}{(\log x-2)^4} 	\\ & =\frac{(4\log x-7)(\log x-2)-2(2\log^2 x-7\log x+5)}{x(\log x-2)^3} 	\\ & =\frac{4\log^2 x-15\log x+14-4\log^2 x+14\log x-10}{x(\log x-2)^3}, 	\end{aligned} \]

che si può riscrivere in modo più compatto come segue:

    \[ 	f''(x)=\frac{4-\log x}{x(\log x-2)^3}. 	\]

Per risolvere f''(x) > 0 per x>0 ed x \neq e^2, consideriamo separatamente il segno del numeratore

    \[ 	4-\log x > 0\implies \log x\le 4\implies x\leq e^4, 	\]

e del denominatore

    \[ 	x(\log x-2)^3>0\implies \log x-2>0\implies \log x>2\implies x>e^2. 	\]

Di conseguenza, f''(x)>0 su (e^2,e^4), dove la funzione è convessa, f''(x)<0 su (0,e^2)\cup(e^4,+\infty), dove la funzione è concava, e si ha un punto di flesso in F=(e^4,5e^4/2).

    \[\quad\]

    \[\quad\]

    \[\quad\]

Rendered by QuickLaTeX.com


 
 

Esercizio 21  (\bigstar\largewhitestar\largewhitestar\largewhitestar\largewhitestar). Studiare le seguenti funzioni e tracciarne un grafico qualitativo:

    \[\quad\]

  1. f(x)=\dfrac{x^2}{x^2-1}
  2.  

  3. f(x)=e^{1/x} x^{1/3}
  4.  

  5. f(x)=6x\log x-(3x-2) \left[\log(3x-2)+1\right]-4\log 2
  6.  

  7. f(x)=x^{1/(4\log^2 x)}

Soluzione punto .

La funzione

    \[ 	f(x) = \frac{x^2}{x^2-1} 	\]

è definita per x^2-1\neq 0, ovvero x \neq \pm 1, per cui il dominio risulta essere:

    \[ 	D=(-\infty,-1)\cup(-1,1)\cup(1,+\infty). 	\]

Dato che f(-x)=f(x), la funzione è pari ed è pertanto sufficiente studiarla in

    \[ 	D_+ = [0,1) \cup (1,+\infty). 	\]

Inoltre interseca gli assi nella sola origine O=(0,0). Nel limite a +\infty sia numeratore che denominatore asintoticamente vanno come x^2, perciò

    \[ 	\lim_{x\to + \infty}f(x)=1, 	\]

mentre nel punto di discontinuità x=1 abbiamo numeratore positivo e denominatore uguale a 0^\pm:

    \[ 	\lim_{x\to^{\pm}}f(x)= \frac{1}{0^\pm} = \pm\infty. 	\]

Di conseguenza, la retta y=1 è un asintoto orizzontale, mentre le rette x=\pm 1 (per parità x=-1 si comporta come x=1 ribaltando i segni) sono asintoti verticali. La derivata prima è data

    \[ 	f'(x)=\frac{2x(x^2-1)-x^2\cdot 2x}{(x^2-1)^2}=-\frac{2x}{(x^2-1)^2}, 	\]

da cui f'(x)=0 se e solo se si annulla il numeratore, ovvero per x=0. Dato che f'(x)>0 per x<0 e f'(x)<0 per x>0, l’origine O=(0,0) è un massimo relativo. La derivata seconda è

    \[ 	f''(x)=-\frac{2(x^2-1)^2-2x\cdot 4x(x^2-1)}{(x^2-1)^4}=\frac{6x^2+2}{(x^2-1)^3}. 	\]

Allora f''(x)\neq 0 per ogni x nel dominio ed f''(x)>0 per x\in(1,+\infty), mentre f''(x)<0 per x\in(0,1). Estendendo l’analisi fatta sin qui per parità, si ottiene il grafico in figura:

    \[\quad\]

    \[\quad\]

    \[\quad\]

Rendered by QuickLaTeX.com

Soluzione punto 2.

La funzione

    \[ 	f(x) = e^{1/x} \sqrt[3]{x} 	\]

è definita per x\neq 0, perciò il dominio è dato da

    \[ 	D=(-\infty,0)\cup(0,+\infty). 	\]

La funzione non interseca gli assi e non presenta simmetrie. I limiti ad infinito sono

    \[ 	\lim_{x\to\pm\infty}f(x)=\pm\infty 	\]

perché e^{1/x} \to 0 e x^{1/3} \to \pm \infty. Il coefficiente angolare di eventuali asintoti obliqui è

    \[ 	\lim_{x\to\pm\infty}\frac{f(x)}{x}= \lim_{x\to\pm\infty} e^{1/x} x^{-2/3} = 0, 	\]

per cui la funzione non presenta asintoti obliqui od orizzontali. Nel punto di discontinuità abbiamo

    \[ 	\lim_{x\to 0^-} f(x)=e^{-\infty}\cdot 0=0, 	\]

mentre da destra si arriva ad una forma indeterminata:

    \[\lim_{x\to 0^+}f(x)=[0]\cdot[\infty]. 	\]

Ponendo t = 1/x il limite per x\to 0^+ corrisponde a t\to +\infty; usando la regola di de l’Hôpital:

    \[\lim_{x\to 0^+} f(x)=\lim_{t\to +\infty}\frac{e^t}{t^{1/3}}=\lim_{t\to+\infty}\frac{e^t}{t^{-2/3}/3}= 	\lim_{t\to+\infty}\frac{1}{3}\cdot e^t\cdot t^{2/3}=+\infty,\]

per cui la retta x=0 è un asintoto verticale destro. La derivata prima si calcola tramite la formula di derivazione per un prodotto:

    \[ 	f'(x)=-\frac{e^{1/x}}{x^2}\cdot x^{1/3}+e^{1/x}\cdot\frac{x^{-2/3}}{3}=e^{1/x}\cdot\frac{x-3}{3x^{5/3}}. 	\]

Per cui f'(x)=0 se e solo se si annulla il numeratore, ovvero per x=3. Dato che f'(x)<0 per x\in(0,3) e f'(x)>0 per x\in(-\infty,0)\cup(3,+\infty), la funzione ha nel punto

    \[ 	m=(3,e^{1/3}\sqrt[3]{3}) 	\]

un minimo relativo. La derivata seconda, invece, è data da

    \[ \begin{aligned} 	f''(x) & = -\frac{e^{1/x}}{x^2}\cdot\frac{x-3}{3x^{5/3}}+e^{1/x}\cdot\frac{x^{5/3}-\dfrac{5}{3}(x-3) x^{2/3}}{3x^{10/3}} 	\\ & =\frac{e^{1/x}}{3x^{11/3}}\cdot\left(-x+3+x^2-\frac{5}{3} x^2+5x\right) 	\\ & =-e^{1/x}\cdot\frac{2x^2-12x-9}{9x^{11/3}}. 	\end{aligned}\]

Si ha f''(x)=0 se e solo se si annulla il numeratore, ovvero

    \[2x^2 -12x - 9 = 0 \iff x=3\pm\frac{3\sqrt{6}}{2},\]

per cui in tali punti la funzione presenta dei flessi. Dato che f''(x)>0 per

    \[x\in\left(3-\frac{3\sqrt{6}}{2},0\right)\cup\left(0,3+\frac{3\sqrt{6}}{2}\right),\]

la funzione risulta convessa su tale intervallo. Il grafico è riportato in figura:

    \[\quad\]

    \[\quad\]

    \[\quad\]

Rendered by QuickLaTeX.com

Soluzione punto 3.

La funzione

    \[ 	f(x) = 6x \log x - (3x - 2) [\log(3x-2) +1] - 4 \log 2 	\]

è definita per x>0 e 3x-2>0. Il dominio è dunque dato da

    \[ 	D=\left(\frac{2}{3},+\infty\right). 	\]

Il limite all’estremo sinistro del dominio è dato da

    \[ 	\lim_{x\to(2/3)^+} f(x)=4\log\frac{2}{3}-4\log 2=-4\log 3<0, 	\]

mentre il limite a +\infty porta ad una forma indeterminata:

    \[\lim_{x\to+\infty} f(x)=\infty-\infty.\]

Si può risolvere applicando de l’Hôpital dopo alcune manipolazioni algebriche:

    \[\begin{aligned} 	\lim_{x\to+\infty}f(x) & =\lim_{x\to+\infty}\left[6x\log x-3x\log 3x\right] 	\\ & =\lim_{x\to+\infty} 3x \left[ 2 \log x - \log 3x \right] =\lim_{x\to+\infty} \frac{3x}{\log x} \left[ 2 - \frac{\log 3x}{\log x} \right] 	\\ & = \underbrace{\lim_{x\to+\infty} \frac{3x}{\log x}}_{=+\infty}  \cdot \lim_{x\to+\infty}  \left[ 2 - \underbrace{\frac{1/x}{1/x}}_{=1} \right] = + \infty. 	\end{aligned}\]

Inoltre, è facile verificare che

    \[\lim_{x\to+\infty}\frac{f(x)}{x}=+\infty,\]

per cui la funzione non ha asintoti orizzontali o obliqui. La derivata prima è data da

    \[f'(x)=6\log x+6-3\left[\log(3x-2)+1\right]-3=6\log x-3\log(3x-2)=3\log\frac{x^2}{3x-2},\]

per cui f'(x)=0 se e solo se si annulla il logaritmo:

    \[ 	\log\frac{x^2}{3x-2}=0\iff \frac{x^2}{3x-2}=1\iff x^2-3x+2=0,\]

le cui soluzioni sono x_1=1 e x_2=2. Dato che f'(x)>0 per (2/3,1)\cup(2,+\infty), la funzione ha un massimo (relativo) ed un minimo (relativo) rispettivamente in

    \[ 	M=(1,-1-4\log 2) \quad \text{e} \quad m=(2,-4). 	\]

Infine, la derivata seconda è

    \[f''(x)=3\cdot\frac{3x-2}{x^2}\cdot\frac{2x(3x-2)-3x^2}{(3x-2)^2}=\frac{3(3x-4)}{x(3x-2)},\]

per cui f''(x)=0 se e solo se x=4/3. La funzione ha allora in F=(4/3,10\log 2-8\log 3-2) un flesso, e f''(x)>0 su (4/3,+\infty), intervallo su cui è convessa. Il grafico è riportato di seguito:

    \[\quad\]

    \[\quad\]

    \[\quad\]

Rendered by QuickLaTeX.com

Soluzione punto 4.

Il dominio della funzione

    \[ 	f(x) = x^{\frac{1}{4 \log^2 x}} 	\]

dipende da quello di \log x, ovvero x>0, e dalla condizione \log x \neq 0 che corrisponde dunque a:

    \[ 	D=(0,1)\cup(1,+\infty). 	\]

Il limite a + \infty è una forma indeterminata del tipo [\infty]^[0], quindi utilizziamo la seguente proprietà di logaritmo ed esponenziale:

    \[ 	\exp \log (g(x)) = g(x). 	\]

Si ha

    \[\ 	\lim_{x\to+\infty}f(x)=\lim_{x\to +\infty}\exp\left[\frac{\log x}{4\log^2 x}\right]=e^{0^+}=1, 	\]

da cui segue che la retta y=1 è un asintoto orizzontale. Un discorso analogo si fa per il limite in 0^+:

    \[ 	\lim_{x\to 0^+}f(x)=[0]^[0]=\lim_{x\to 0^+}\exp\left[\frac{\log x}{4\log^2 x}\right]=e^{0}=1. 	\]

Infine, nel punto di discontinuità x=1 facciamo i limiti da sinistra e destra

    \[ \begin{aligned} 	& \lim_{x\to 1^-}f(x)=\lim_{x\to 1^-}\exp\left[\frac{\log x}{4\log^2 x}\right]=e^{-\infty}=0^+, 	\\ & \lim_{x\to 1^+}f(x) =\lim_{x\to 1^+}\exp\left[\frac{\log x}{4\log^2 x}\right]=e^{+\infty}=+\infty, 	\end{aligned}\]

da cui deduciamo che la retta x=1 è un asintoto verticale destro. La derivata prima è data da

    \[f'(x)=\left(\exp\left[\frac{1}{4\log x}\right]\right)'=\exp\left[\frac{1}{4\log x}\right]\cdot\frac{-1}{4x\log^2 x},\]

perciò si vede subito che f'(x) < 0 sul dominio D, ovvero f è strettamente decrescente. Infine osserviamo che la funzione f può essere scritta come

    \[ 	f(x)=e^{g(x)},\qquad g(x)=\frac{1}{4\log x}, 	\]

per cui la derivata seconda è data dalla formula:

    \[ 	f'(x)=e^{g(x)}\cdot g'(x) \implies f''(x)=e^{g(x)}\cdot[g'(x)]^2+e^{g(x)}\cdot g''(x)=e^{g(x)}\left([g'(x)]^2+g''(x)\right), 	\]

dove

    \[ 	g'(x)=-\frac{1}{4x\log^2 x}, \qquad g''(x)=\frac{\log x+2}{4x^2\log^3 x}. 	\]

Sostituendo g' e g'' nella formula sopra troviamo

    \[f''(x)=\exp\left[\frac{1}{4\log x}\right]\left(\frac{1}{16x^2\log^4 x}+\frac{\log x+2}{4x^2\log^3 x}\right)\]

che, facendo alcune manipolazioni algebriche, si può riscrivere più compattamente come segue:

    \[ 	f''(x)=\exp\left[\frac{1}{4\log x}\right]\cdot\frac{4\log^2 x+8\log x+1}{16x^2\log^4 x}. 	\]

Ne segue che f''(x)=0 se e solo se si annulla il numeratore, ovvero

    \[4\log^2 x+8\log x+1=0\iff \log x=-1\pm\frac{\sqrt{3}}{2},\]

e quindi per x=e^{-1\pm\frac{\sqrt{3}}{2}}, punti in cui la funzione presenta due flessi come si vede dal grafico seguente:

    \[\quad\]

    \[\quad\]

    \[\quad\]

Rendered by QuickLaTeX.com


 
 

Esercizio 22  (\bigstar\bigstar\bigstar\largewhitestar\largewhitestar). Studiare le seguenti funzioni e tracciarne un grafico qualitativo:

    \[\quad\]

  1. f(x)=1-e^{(1+\sin x)/(\sin x-1)}
  2.  

  3. f(x)=\dfrac{2^{x+1}+1}{2^x-1}
  4.  

  5. f(x)=\arctan\left(\dfrac{4x-1}{4x}-\dfrac{1}{4|x|}\right)
  6.  

  7. f(x)=\log\left(\sqrt{x^2+1}-x\right)-2\arctan\dfrac{1}{x}

Soluzione punto 1.

La funzione

    \[ 	f(x) = 1 - \exp \left( \frac{\sin x + 1}{\sin x - 1}\right) 	\]

risulta periodica di periodo T=2\pi perciò ci limitiato all’intervallo [0,2\pi). La funzione è definita per

    \[ 	\sin x \neq 1 \iff x = \pi/2, 	\]

e quindi il dominio risulta essere

    \[ 	D=\left[0,\frac{\pi}{2}\right)\cup\left(\frac{\pi}{2},2\pi\right).\]

Si ha f(0)=1-e^{-1}, perciò la funzione passa per il punto A=(0,1-1/e). Inoltre

    \[ 	\lim_{x\to(\pi/2)^{\pm}} f(x)=1-e^{-\infty}=1, 	\]

per cui la funzione presenta una discontinuità eliminabile in x=\pi/2 ponendo f(\pi/2)=1. Si noti che lo studio di asintoti orizzontali o obliqui non ha senso trattandosi di una funzione periodica.

Passiamo a calcolare la derivata prima sfruttando la formula di differenziabilità per una composizione; precisamente, si trova

    \[ 	\begin{aligned} f'(x) & = -e^{(1+\sin x)/(\sin x-1)}\cdot\frac{\cos x(\sin x-1)-\cos x(\sin x+1)}{(\sin x-1)^2} 	\\ & =\frac{2\cos x\cdot e^{(1+\sin x)/(\sin x-1)}}{(\sin x-1)^2}, 	\end{aligned}\]

per cui f'(x)=0 se e solo se x_1=\pi/2 o x_2=3\pi/2. La funzione non è definita in x_1, mentre x_2 risulta coincidere con il punto di minimo m=(3\pi/2,0). Infine, osserviamo che la funzione f può essere scritta come

    \[ 	f(x)=1-e^{g(x)},\qquad g(x)=\frac{1+\sin x}{\sin x-1}, 	\]

per cui

    \[f'(x)=-e^{g(x)}\cdot g'(x),\]

e questo ci aiuta nel calcolo della derivata seconda grazie alla formula seguente:

    \[ 	f''(x)=-e^{g(x)}\cdot[g'(x)]^2-e^{g(x)}\cdot g''(x)=-e^{g(x)}\left([g'(x)]^2+g''(x)\right). 	\]

Si verifica facilmente che

    \[ 	g'(x)=\frac{-2\cos x}{(\sin x-1)^2},\qquad g''(x)=\frac{-2(\sin x+2)}{(\sin x-1)^2}, 	\]

quindi sostituendo nella formula sopra si arriva alla derivata seconda di f:

    \[ 	\begin{aligned} f''(x) & = -e^{(\sin x+1)/(\sin x-1)}\cdot\frac{4\cos^2 x-2(\sin x+2)(\sin x-1)^2}{(\sin x-1)^4} 	\\ & =-e^{(\sin x+1)/(\sin x-1)}\cdot\frac{4\cos^2 x-2\sin^3 x+4\sin^2 x-2\sin x-4\sin^2 x+8\sin x-4}{(\sin x-1)^2}. 	\end{aligned}\]

Facendo alcune simplificazioni si trova l’espressione

    \[f''(x)=e^{(\sin x+1)/(\sin x-1)}\cdot\frac{2\sin x(\sin^2 x+2\sin x-3)}{(\sin x-1)^2},\]

da cui segue che f''(x)=0 se e solo se si annulla il numeratore. In particolare, o si ha

    \[ 	\sin x=0\iff x=0,\pi 	\]

oppure si annulla il secondo fattore:

    \[ 	\sin^2 x+2\sin x-3=0. 	\]

Ponendo t=\sin x si arriva alle soluzioni t=-3 e t=1, entrambe non risolvibili sul dominio di f. Ne segue che la funzione presenta punti di flesso in

    \[ 	A=(0,1-1/e),\qquad B=(\pi,1-1/e),\qquad C=(2\pi,1-1/e). 	\]

Il grafico è riportato in figura:

    \[\quad\]

    \[\quad\]

    \[\quad\]

Rendered by QuickLaTeX.com

Soluzione punto 2.

La funzione

    \[ 	f(x) = \frac{2^{x+1}+1}{2^x-1} 	\]

è definita quando il denominatore è diverso da zero, ovvero

    \[ 	2^x - 1 \neq 0 \iff 2^x \neq 1 \iff x \neq 0. 	\]

Il dominio risulta dunque essere

    \[ 	D=(-\infty,0)\cup(0,+\infty). 	\]

I limiti a \pm \infty si calcola banalmente ricordando che 2^{x} \to 0 quando x \to - \infty; precisamente, si trova che

    \[ 	\lim_{x\to-\infty}f(x)=\frac{0+1}{0-1}=-1,\qquad \lim_{x\to+\infty} f(x)=\lim_{x\to+\infty}\frac{2^{x+1}}{2^x}=2,\]

per cui le rette y=-1 e y=2 sono asintoti orizzontali a sinistra e destra rispettivamente. Inoltre, i limiti da destra e sinistra nel punto di discontinuità sono

    \[ 	\lim_{x\rightarrow 0^{\pm}}f(x)=\pm\infty, 	\]

per cui la retta x=0 è un asintoto verticale. La derivata prima è data da

    \[ 	f'(x)=\frac{2^{x+1}\log 2(2^x-1)-(2^{x+1}-1)2^x\log 2}{(2^x-1)^2}=\frac{-2^x\log 2}{(2^x-1)^2}, 	\]

ed è sempre negativa, perciò f è decrescente nel dominio. La derivata seconda è data da

    \[f''(x)=-\log 2\cdot\frac{2^x\log 2(2^x-1)^2-2^x\cdot2(2^x-1)2^x\log 2}{(2^x-1)^4},\]

che può essere riscritta in maniera più compatta come segue:

    \[ 	f''(x)=\frac{2^x\log^2 2\cdot(2^{x+1}-2^x+1)}{(2^x-1)^3}. 	\]

Allora f''(x)=0 se e solo se

    \[2^{x+1}-2^x-1=0\sesolose 2^x-1=0,\]

la cui unica soluzione è x=0, che però non appartiene al dominio. Dunque, la funzione risulta convessa su (0,+\infty) e concava su (-\infty,0). Il grafico è riportato in figura:

    \[\quad\]

    \[\quad\]

    \[\quad\]

Rendered by QuickLaTeX.com

Soluzione punto 3.

Il dominio della funzione

    \[ 	f(x) = \arctan \left( \frac{4x-1}{4x} - \frac{1}{4|x|} \right) 	\]

si trova imponendo i denominatori diversi da zero, ovvero x \neq 0:

    \[ 	D=(-\infty,0)\cup(0,+\infty). 	\]

La funzione si può scrivere sviluppando il modulo |x| come

    \[ 	f(x) = \begin{cases} f_1(x) & \text{se } x > 0, \\ f_2(x) & \text{se } x < 0,\end{cases} 	\]

dove

    \[ 	\begin{aligned} & f_1(x)=\arctan\frac{2x-1}{2x}, 	\\ & f_2(x)=\arctan 1= \pi/4. 	\end{aligned}\]

In particolare, la funzione risulta costante sull’intervallo (-\infty,0). Il limite a -\infty coincide con il valore della funzione su (-\infty, 0), quindi ci occupiamo solo del limite a +\infty:

    \[ 	\lim_{x\to +\infty} f_1(x)=\arctan 1=\frac{\pi}{4}, 	\]

dal momento che (2x-1)/2x \sim 2x / 2x asintoticamente quando x è sufficientemente grande. Inoltre, si vede che

    \[ 	\lim_{x\to 0^+} f(x)= \lim_{x\to 0} f_1(x) = \arctan(-\infty)=-\frac{\pi}{2}, 	\]

mentre da sinistra

    \[ 	\lim_{x\to 0^-} f(x)= \lim_{x\to 0} f_2(x) = \frac{\pi}{4}, 	\]

per cui la retta y=\pi/4 è un asintoto orizzontale destro mentre la funzione presenta una discontinuità di prima specie in x=0 con salto s=3\pi/4. La derivata è nulla in (-\infty,0) e

    \[ \begin{aligned} 	f_1'(x) & =\frac{1}{\displaystyle 1+\left(\frac{2x-1}{2x}\right)^2}\cdot\frac{4x-4x+2}{4x^2} 	\\ & = \frac{4x^2}{4x^2+4x^2+1-4x}\cdot\frac{1}{2x^2} 	\\ & =\frac{2}{8x^2-4x+1} 	\end{aligned}\]

in (0,+\infty). Dunque, si ha f_1'(x)>0 su tale intervallo e, di conseguenza, la funzione è sempre crescente (debolmente in (-\infty,0)). Infine

    \[ 	f_1''(x)=-\frac{2(16x-4)}{(8x^2-4x+1)^2}=-\frac{8(4x-1)}{(8x^2-4x+1)^2}, 	\]

per cui f_1''(x)=0 se e solo se x=1/4, che appartiene al dominio di f_1, da cui segue che la funzione ha un flesso nel punto F=(1/4,-\pi/4) e risulta convessa per x\in(0,1/4) e concava per x\in(1/4,+\infty). Il grafico è riportato in figura:

    \[\quad\]

    \[\quad\]

    \[\quad\]

Rendered by QuickLaTeX.com

Soluzione punto 4.

La funzione

    \[ 	f(x) = \log \left( \sqrt{x^2+1} - x \right) - 2 \arctan \frac1x 	\]

è definita per x \neq 0 e argomento del logaritmo positivo:

    \[ 	\sqrt{1+x^2}-x>0. 	\]

Possiamo portare x a destra ed elevare al quadrato, ottenendo la disequazione

    \[ 	1 + x^2 > x^2, 	\]

che è sempre banalmente verificata. In particolare, il dominio è dato da:

    \[ 	D=(-\infty,0)\cup(0,+\infty). 	\]

Il limite a - \infty si calcola osservando che il termine di logaritmo esplode:

    \[\lim_{x\to-\infty}f(x)=+\infty,\]

perciò non c’è asintoto orizzontale sinistro. Inoltre, il coefficiente angolare di un eventuale asintoto obliquo sinistro è dato da:

    \[ 	m=\lim_{x\to-\infty}\frac{f(x)}{x}=\lim_{x\to-\infty}\frac{\log(\sqrt{1+x^2}-x)}{x}= 	\lim_{x\to-\infty}\frac{\log(-2x)}{x}=0, 	\]

perciò a sinistra non ammette asintoti di alcun tipo. Il limite a + \infty è più delicato, ma si può calcolare osservando che

    \[ 	\sqrt{x^2+1} - x = \sqrt{x^2+1} - x \frac{\sqrt{x^2+1} + x}{\sqrt{x^2+1} + x} = \frac{1}{\sqrt{x^2+1} + x}, 	\]

da cui segue che:

    \[\lim_{x\to+\infty}f(x)=\lim_{x\to+\infty}\log\frac{1}{\sqrt{1+x^2}+x}=-\infty.\]

Inoltre, il coefficiente angolare di un eventuale asintoto obliquo destro è dato da

    \[ 	m=\lim_{x\to+\infty}\frac{f(x)}{x}=\lim_{x\to+\infty}\frac{1}{x} \log\frac{1}{\sqrt{1+x^2}+x}=0,\]

da cui deduciamo che non vi sono asintoti orizzontali/obliqui. Inoltre, nel punto di discontinuità x=0 abbiamo

    \[ 	\lim_{x\to 0^{\pm}}f(x)=0-2\arctan(\pm\infty)=\mp\pi,\]

per cui la discontinuità di prima specie in x=0 con salto -2\pi. La derivata prima è data da

    \[ \begin{aligned} 	f'(x) & =\frac{\dfrac{x}{\sqrt{1+x^2}}-1}{\sqrt{1+x^2}-x}-2\frac{1}{1+\dfrac{1}{x^2}}\left(-\frac{1}{x^2}\right)= 	-\frac{1}{\sqrt{1+x^2}}+\frac{2}{1+x^2} 	\\[.4em] & =\frac{2\sqrt{1+x^2}-x^2-1}{\sqrt{(1+x^2)^3}}. 	\end{aligned} \]

Dunque f'(x)=0 se e solo se si annulla il numeratore, ovvero:

    \[ 	2\sqrt{1+x^2}-x^2-1=0 \iff 2(1+x^2)=(x^2+1)^2\iff  x^4-1=0\iff x=\pm 1. 	\]

Dato che f'(x)>0 per (-1,0)\cup(0,1), f ammette un massimo in M=(-1,\log(\sqrt{2}+1)+\pi/2) ed un minimo in m = (1,\log(\sqrt{2}-1)-\pi/2), entrambi relativi. Infine, potendo scrivere

    \[ 	f'(x)=\frac{2\sqrt{g(x)}-g(x)}{\sqrt{[g(x)]^3}}=\frac{2}{g(x)}-\frac{1}{\sqrt{g(x)}},\qquad g(x)=x^2+1, 	\]

la derivata seconda si calcola come segue:

    \[\begin{aligned} 	f''(x) & = -\frac{2g'(x)}{[g(x)]^2}+\frac{g'(x)}{2\sqrt{[g(x)]^3}}=\frac{g'(x)(\sqrt{g(x)}-4)}{2[g(x)]^2} 	\\[.4em] & =\frac{x(\sqrt{1+x^2}-4)}{(1+x^2)^2}. 	\end{aligned} \]

Allora f''(x)=0 se e solo se si annulla il numeratore, ovvero x=0 (che non è accettabile) oppure

    \[ 	\sqrt{1+x^2}-4=0\iff 1+x^2=16\iff x=\pm\sqrt{15}, 	\]

per cui la funzione ammette due flessi in tali punti. La funzione è perciò convessa su (-\sqrt{15},0)\cup(\sqrt{15}+\infty). Il grafico della funzione è riportato in figura:

    \[\quad\]

    \[\quad\]

    \[\quad\]

Rendered by QuickLaTeX.com


 
 

Esercizio 23  (\bigstar\bigstar\bigstar\bigstar\largewhitestar). Studiare le seguenti funzioni e tracciarne un grafico qualitativo:

    \[\quad\]

  1. f(x)=\dfrac{1+|\log x|}{1-|\log x|}
  2.  

  3. f(x)=x+\log\cosh x-\tanh x
  4.  

  5. f(x)=\arcsin\sqrt{1-x^2}+\sqrt{1-x^2}
  6.  

  7. f(x)=\dfrac{2-\log^{-1} x}{\sqrt{|\log x|}}

Soluzione punto 1.

La funzione

    \[ 		f(x)= \frac{1 + |\log x|}{1-|\log x|} 		\]

è definita per x>0 e |\log x|\neq 1, ovvero

    \[ 		\log x \neq \pm 1 \iff x \neq 1/e \text{ e } x \neq e, 		\]

da cui si arriva al dominio:

    \[ 		D=\left(0,\frac{1}{e}\right)\cup\left(\frac{1}{e},e\right)\cup\left(e,+\infty\right). 		\]

Sviluppando il modulo di \log x si può scrivere la funzione come

    \[ 		f(x) = \begin{cases} f_1(x) & \text{se } x \in [1,e) \cup (e,+\infty) \\ f_2(x) & \text{se } x \in (0,1/e) \cup (1/e,1), \end{cases} 		\]

dove

    \[  		f_1(x)=\frac{1+\log x}{1-\log x} \quad \text{e} \quad f_2(x)=\frac{1-\log x}{1+\log x}. 		\]

Numeratore e denominatore vanno ad infinito con la stessa velocità per x \to + \infty, perciò

    \[ 		\lim_{x\to+\infty}f(x)=\lim_{x\to+\infty}\frac{1/x}{-1/x}=-1, 		\]

applicando, ad esempio, il teorema di de l’Hôpital. Inoltre, si ha

    \[\lim_{x\rightarrow 0^+}f(x)=\lim_{x\rightarrow 0^+}\frac{-1/x}{1/x}=-1,\]

per cui la funzione presenta una discontinuità eliminabile in x=0 ponendo f(0)=-1. Infine

    \[ \begin{aligned} 		& \lim_{x\rightarrow(1/e)^-}f(x)=-\infty,\qquad \lim_{x\rightarrow(1/e)^+}f(x)=+\infty, 		\\ & \lim_{x\rightarrow e^-}f(x)=+\infty,\qquad \lim_{x\rightarrow e^+}f(x)=-\infty, 		\end{aligned}\]

per cui le rette x=1/e ed x=e sono asintoti verticali, mentre y=-1 un asintoto orizzontale. Per calcolare la derivata prima di f occupiamoci dei due rami f_1 ed f_2 separatamente:

    \[ 		\begin{aligned} & f_1'(x)=\frac{\dfrac{1}{x}(1-\log x)+\dfrac{1}{x}(1+\log x)}{(1-\log x)^2}=\frac{2}{x(1-\log x)^2}, 		\\ & f'_2(x)=\frac{-\dfrac{1}{x}(1+\log x)-\dfrac{1}{x}(1-\log x)}{(1+\log x)^2}=-\frac{2}{x(1+\log x)^2}, 		\end{aligned} \]

per cui la funzione è crescente su (1,e)\cup(e,+\infty) e decrescente su (0,1/e)\cup(1/e,1). Di conseguenza, m=(1,1) è un punto di minimo relativo. Infine, si ha

    \[ 		\begin{aligned} & f_1''(x)=-2\frac{(1-\log x)^2-2x(1-\log x)\dfrac{1}{x}}{x^2(1-\log x)^4}=-\frac{2(\log x+1)}{x^2(\log x-1)^3}, 		\\ & f_2''(x)=2\frac{(1+\log x)^2+2x(1+\log x)\dfrac{1}{x}}{x^2(\log x+3)^4}=\frac{2(\log x+3)}{x^2(\log x+1)^3}, 		\end{aligned}\]

per cui f_1''(x)=0 per x=1/e che non appartiene al dominio, mentre f_2''(x)=0 per x=1/e^3. La funzione ha allora un flesso nel punto F=(1/e^3,-2), risulta convessa su (0,1/e^3)\cup(1/e,e) e concava su (1/e^3,1/e)\cup(e,+\infty). Il grafico è riportato in figura:

    \[\quad\]

    \[\quad\]

    \[\quad\]

Rendered by QuickLaTeX.com

Soluzione punto 2.

La funzione

    \[ 		f(x) = x + \log \cosh x - \tanh x 		\]

è definita quando \cosh x>0, ovvero su tutto \mathbb R. Inoltre f(0)=0, quindi la funzione passa per l’origine degli assi. Il limite a meno infinito è una forma indeterminata:

    \[ 		\lim_{x\to-\infty}f(x)=[\infty] - [\infty], 		\]

perciò sfruttiamo il fatto che \log ( e^x + e^{-x}) è asintoticamente equivalente a \log( e^{-x}) = -x per x \to - \infty, ottenendo:

    \[ 		\lim_{x\to-\infty}f(x)=\lim_{x\to-\infty}\left(x+(-x)-\log 2+1\right) =\lim_{x\to-\infty}\left(x-x-\log 2+1\right)=-\log 2+1. 		\]

Il limite a + \infty, invece, non risulta in una forma indeterminata e si ha:

    \[ 		\lim_{x\to+\infty}f(x)=+\infty. 		\]

Cerchiamo eventuali asintoti obliqui destri; il coefficiente angolare a + \infty è dato da

    \[ 		m_+=\lim_{x\to+\infty}\frac{f(x)}{x}=\lim_{x\to+\infty}\left(1+\frac{\log\cosh x}{x}-\frac{\tanh x}{x}\right)=1+1-0=2, 		\]

mentre il termine noto è

    \[ 		q=\lim_{x\to+\infty}[f(x)-m_+ x]=\lim_{x\to+\infty}\left(-x+\log(e^x+e^{-x})-\log 2-1\right) =-\log 2-1. 		\]

Di conseguenza, la retta y=-\log 2+1 è un asintoto orizzontale sinistro, mentre la retta y=2x-\log 2-1 è un asintoto obliquo destro. Per la derivata prima si ha

    \[f'(x)=1+\frac{\sinh x}{\cosh x}-(1-\tanh^2 x)=\tanh x(\tanh x+1)\]

usando la relazione \sinh(x) / \cosh(x) = \tanh(x). Osserviamo perciò che

    \[ 		f'(x) = 0 \iff \tanh(x) = 0 \text{ oppure } \tanh(x) = -1, 		\]

per cui x=0 è la soluzione della prima, mentre per la seconda si ha:

    \[ 		\tanh x=-1\iff \frac{e^{2x}-1}{e^{2x}+1}=-1\iff e^{2x}-1=-e^{2x}-1, 		\]

ovvero non ha soluzione. Dato che f'(x)>0 per x>0, il punto O=(0,0) è un minimo assoluto della funzione. La derivata seconda è data da

    \[f''(x)=2\tanh x(1-\tanh^2 x)+(1-\tanh^2 x)=(1-\tanh^2 x)(2\tanh x+1),\]

e quindi f''(x)=0 in uno dei seguenti casi:

    \[1-\tanh^2 x=0\iff \tanh x=\pm 1\iff e^{2x}-1=\pm(e^{2x}+1),\]

che non ammette soluzione, oppure

    \[ 		2\tanh x+1=0\iff 2(e^{2x}-1)=-(e^{2x}+1)\iff 3e^{2x}=1\iff x=-\frac{1}{2}\log 3<0. 		\]

La funzione presenta pertanto un flesso nel punto F=(-(\log 3)/2, \log(2/3)+1/2) e risulta essere convessa su (-(\log 3)/2,+\infty) e concava su (-\infty,-(\log 3)/2). Il grafico è rappresentato in figura:

    \[\quad\]

    \[\quad\]

    \[\quad\]

Rendered by QuickLaTeX.com

Soluzione punto 3.

La funzione

    \[ 		f(x) = \arcsin \sqrt{1-x^2} + \sqrt{1-x^2} 		\]

è definita per 0\leq \sqrt{1-x^2}\leq 1, e quindi ha per dominio

    \[ 		D=[-1,1]. 		\]

Inoltre, è facile verificare che si tratta di una funzione pari, perciò possiamo limitarci a studiarla nell’intervallo D_+=[0,1]. Poiché f(1)=0 e f(0)=\pi/2+1, la funzione passa per i punti

    \[ 		A=(-1,0),\qquad B=(0,\pi/2+1),\qquad C=(1,0). 		\]

Non dobbiamo calcolare alcun limite, perciò partiamo con il calcolo della derivata prima:

    \[ \begin{aligned} 		f'(x)&=\frac{1}{\sqrt{1-(\sqrt{1-x^2})^2}}\cdot\frac{-2x}{2\sqrt{1-x^2}}+\frac{-2x}{2\sqrt{1-x^2}} 		\\& = -\frac{x}{\sqrt{1-x^2}}\left(\frac{1}{|x|}+1\right). 		\end{aligned}\]

La derivata è definita in (-1,1) \setminus \{0\}. Calcolando il limite da sinistra e destra rispettivamente in \pm 1 otteniamo:

    \[ 		\lim_{x\to 1^-}f'(x)=- \infty \quad \text{e} \quad \lim_{x\to -1^+}f'(x)= \infty, 		\]

da cui segue che in tali punti si ha una cuspide. Inoltre, in x=0 si ha

    \[ 		\lim_{x\to 0^{\pm}} f'(x)=\lim_{x\to 0^{\pm}}-\frac{x}{|x|}=\mp 1, 		\]

perciò x=0 è un punto angoloso per f. La funzione risulta crescente su [-1,0) e decrescente in (0,1], per cui ha due minimi assoluti nei punti A,\ C e un massimo assoluto nel punto B.

Infine, sviluppando il modulo e ponendo

    \[ 		f'_{\pm}(x)=\frac{\pm 1-x}{\sqrt{1-x^2}}, 		\]

la derivata seconda è data da:

    \[f''_{\pm}(x)=\frac{-\sqrt{1-x^2}-(\pm 1-x)\cdot\dfrac{-x}{\sqrt{1-x^2}}}{1-x^2} = \frac{\pm x-1}{\sqrt{(1-x^2)^3}}. 		\]

E’ immediato verificare che f''(x)=0 se e solo se x=\pm 1, i quali non possono essere punti di flesso, e f''(x)<0 su tutto il dominio. Di conseguenza, la funzione è sempre concava e, sfruttandone la simmetria pari, si ottiene il grafico è riportato in figura:

    \[\quad\]

    \[\quad\]

    \[\quad\]

Rendered by QuickLaTeX.com

Soluzione punto 4.

La funzione

    \[ 		f(x) = \frac{2 + \dfrac{1}{\log x}}{\sqrt{|\log x|}} 		\]

è definita per x>0 e |\log x|\neq 0, per cui

    \[ 		D=(0,1)\cup(1,+\infty). 		\]

Sviluppiamo il modulo e scriviamo la funzione come

    \[ 		f(x) = \begin{cases} f_1(x) & \text{se } x \in (1,+\infty), \\ f_2(x) & \text{se } x \in (0,1),\end{cases} 		\]

dove:

    \[ 		f_1(x)=\frac{2\log x-1}{\sqrt{\log^3 x}} \quad \text{e} \quad f_2(x)=\frac{2\log x-1}{\sqrt{-\log^3 x}}. 		\]

I limiti si svolgono in maniera banale osservando che in 0^+ e + \infty il logaritmo a denomimatore è il termine dominante, mentre in 1^\pm abbiamo -1/0^+; riassumendo:

    \[ \begin{aligned} & \lim_{x\rightarrow 0^+}f(x)=0^+, 		\\ & \lim_{x\rightarrow 1^{\pm}}f(x)=-\infty, 		\\ & \lim_{x\rightarrow +\infty}f(x)=0^+. 		\end{aligned} \]

La funzione ha dunque una discontinuità eliminabile in x=0 (ponendo f(0)=0), ammette la retta x=1 come asintoto verticale e la retta y=0 come asintoto orizzontale destro. La derivata è

    \[ 		f_1'(x)=\frac{\dfrac{2}{x}\cdot\sqrt{\log^3 x}-(2\log x-1)\cdot\dfrac{1}{2\sqrt{\log^3 x}}\cdot\dfrac{3\log^2 x}{x}}{\log^3 x},\]

che possiamo riscrivere più comodamente come segue:

    \[f_1'(x)=\frac{3-2\log x}{2x\sqrt{\log^5 x}}.\]

In particolare, f_1'(x)=0 se e solo se 3-2\log x=0 da cui x=e^{3/2}; dunque, la funzione f_1 è crescente su (1,e^{3/2}) ed ammette un massimo nel punto M=(e^{3/2},4\sqrt{6}/9). Analogamente, la derivata del secondo ramo è

    \[ 		f_2'(x)=\frac{\dfrac{2}{x}\cdot\sqrt{-\log^3 x}-(2\log x-1)\cdot\dfrac{1}{2\sqrt{-\log^3 x}}\cdot\dfrac{-3\log^2 x}{x}}{-\log^3 x}, 		\]

che si riscrive più comodamente come segue:

    \[ 		f_2'(x)=\frac{3-2\log x}{2x\sqrt{-\log^5 x}}. 		\]

Questa funzione è strettamente positiva nel dominio di definizione, perciò f_2 è sempre crescente. Infine, la derivata seconda è data da

    \[ 		f''_1(x)=\frac{4\log^2 x-15}{4x^2\sqrt{\log^7 x}}\quad  \text{e} \quad f_2''(x)=\frac{15-4\log^2 x}{4x^2\sqrt{-\log^7 x}},\]

che si annullano entrambe quando il numeratore (che è lo stesso a meno del segno) si annulla, ovvero:

    \[ 		4\log^2 x-15=0\iff \log x=\pm\frac{\sqrt{15}}{2}\iff x=e^{\pm\sqrt{15}/2}. 		\]

La prima soluzione è accettabile per f_1, mentre la seconda per f_2. Perciò, la funzione f ha due flessi in tali punti ed inoltre:

    \[\quad\]

  • f_2 è convessa su (0,e^{-\sqrt{15}/2});
  •  

  • f_1 è convessa su (e^{\sqrt{15}/2},+\infty).

Il grafico è riportato in figura.

    \[\quad\]

    \[\quad\]

    \[\quad\]

Rendered by QuickLaTeX.com


 
 

Esercizio 24  (\bigstar\bigstar\bigstar\largewhitestar\largewhitestar). Studiare le seguenti funzioni e tracciarne un grafico qualitativo:

    \[\quad\]

  1. f(x)=|x^2-2x|e^x
  2.  

  3. f(x)=x^2 e^{\dfrac{|x|-1}{|x|}}
  4.  

  5. f(x)=\sin x \cdot e^{|\sin x|}
  6.  

  7. f(x)=\arctan\dfrac{\log x-1}{\log x+1}+\log(\log^2 x+1)

Soluzione punto 1.

La funzione

    \[     f(x) = |x^2-2x| e^x     \]

risulta definita su tutto l’insieme dei numeri reali. Sviluppando il modulo, possiamo riscriverla come segue:

    \[     f(x) = \begin{cases} (x^2-2x) e^x & \text{per } x\in(-\infty,0]\cup(2,+\infty),     \\ (2x-x^2) e^x & \text{per } x\in(0,2]. \end{cases}     \]

La funzione interseca l’asse delle y nell’origine (dato che f(0)=0) e l’asse delle x in

    \[     f(x) = 0 \iff x^2-2x=0 \iff x = 0 \text{ o } x = 2,     \]

ovvero nell’origine e nel punto A=(2,0). Inoltre, la funzione f è non-negativa su tutto \mathbb R essendo prodotto di due funzioni non-negative. Il limite a +\infty è banalmente dato da

    \[     \lim_{x\to +\infty}f(x)=+\infty,     \]

mentre a -\infty abbiamo una forma indeterminata

    \[     \lim_{x\to-\infty}f(x)= [+\infty] \cdot [0].     \]

Tuttavia, la funzione esponenziale è più veloce di ogni polinomio e dunque

    \[     \lim_{x\to-\infty}f(x) = 0.     \]

In particolare, la retta y=0 è un asintoto orizzontale sinistro. Osserviamo inoltre che

    \[     \lim_{x\to +\infty}\frac{f(x)}{x}=+\infty,     \]

quindi la funzione non ammette asintoto orizzontale oppure obliquo a destra. Calcoliamo la derivata dei due “rami” della funzione separatamente, ovvero

    \[f'_1(x)=(2x-2)e^x+(x^2-2x)e^x=(x^2-2)e^x,\]

    \[f'_2(x)=(2-2x)e^x+(2x-x^2)e^x=(2-x^2)e^x,\]

da cui risulta che

    \[ f'(x) =  \begin{cases} (x^2-2) e^x & \text{per } x\in(-\infty,0)\cup(2,+\infty),     \\ (2-x^2) e^x & \text{per } x\in(0,2). \end{cases} \]

In questo caso, i punti x=0 ed x=2 vanno analizzati separatamente. Osserviamo subito che

    \[ \lim_{x \to 0^-} f'(x) = f_1'(0) = (0-2) = -2, \]

mentre

    \[ \lim_{x \to 0^+} f'(x) = f_2'(0) = (2-0) = 2, \]

perciò x=0 risulta essere un punto angoloso. Analogamente, si ha

    \[ \lim_{x \to 2^-} f'(x) = f_2'(2) = (4-4)e^2 = 0, \]

mentre

    \[ \lim_{x \to 2^+} f'(x) = f_1'(2) = (4-4)e^2 = 0, \]

perciò x=2 è un punto di derivabilità per f. Inoltre

    \[ f'(x) = 0 \iff x=\pm \sqrt2, \]

dunque la funzione cresce su (-\infty,-\sqrt{2})\cup (0,\sqrt2) \cup (2,+\infty) e decresce su (-\sqrt{2},0) \cup (\sqrt{2},2). La funzione presenta quindi due massimi relativi nei punti

    \[ M_1 = (-\sqrt{2},(2+2\sqrt{2})e^{-\sqrt{2}}) \quad \text{e} \quad M_2=(\sqrt{2},(2\sqrt{2}-2)e^{\sqrt{2}}) \]

ed due minimi assoluti nei punti in cui si annulla, ovvero l’origine ed A=(2,0). Infine, calcoliamo la derivata seconda dei due “rami” separatamente:

    \[f''_1(x)=2x e^x+(x^2-2)e^x=(x^2+2x-2)e^x,\]

    \[f''_2(x)=-2x e^x+(2-x^2)e^x=-(x^2+2x-2)e^x.\]

La derivata seconda di f si annulla dunque in

    \[x^2+2x-2=0\iff x=-1\pm\sqrt{3}.\]

La funzione è convessa su (-\infty,-1-\sqrt{3})\cup (0,-1+\sqrt{3}) \cup (2,+\infty) e concava altrove. Inoltre, la funzione presenta due flessi nei punti

    \[F_1=(-1-\sqrt{3},(6+4\sqrt{3})e^{-1-\sqrt{3}})\quad  \text{e} \quad F_2=(-1+\sqrt{3},(4\sqrt{3}-6)e^{-1+\sqrt{3}}).\]

Il grafico è riportato nella figura seguente:

    \[\quad\]

    \[\quad\]

    \[\quad\]

Rendered by QuickLaTeX.com

Soluzione punto 2.

La funzione

    \[ f(x) = x^2 e^{ \frac{|x|-1}{|x|}} \]

è definita per |x|\neq 0, ovvero il dominio è dato da

    \[ D=(-\infty,0)\cup(0,+\infty). \]

Sviluppando il modulo |x|, possiamo riscrivere la funzione come segue:

    \[ f(x) = \begin{cases}x^2 e^{\frac{x-1}{x}} & \text{per } x > 0, \\ x^2 e^{\frac{x+1}{x}} & \text{per } x < 0. \end{cases} \]

Dato che la funzione è pari (ovvero, f(-x)=f(x)), possiamo limitarci a studiare uno dei due rami, ad esempio

    \[ f_1(x) := x^2 e^{\frac{x-1}{x}} \qquad \text{per } x \in (0,+\infty). \]

Il limite dell’esponente per x\to +\infty si calcola facilmente essendo rapporto tra polinomi:

    \[ \lim_{x \to + \infty} \frac{x-1}{x}=1, \]

da cui segue che

    \[ \lim_{x\to + \infty} f(x) = \lim_{x\to + \infty} f_1(x) = \lim_{x\to + \infty} x^2 = + \infty. \]

In particolare, la funzione non ammette asintoti orizzontali. Un conto simile ci permette di affermare che non esistono neanche asintoti obliqui:

    \[ \lim_{x\to + \infty} \frac{f_1(x)}{x} = \lim_{x\to + \infty} x e^{\frac{x - 1}{x}} = \lim_{x \to + \infty} x = \infty. \]

Il limite da destra in zero si svolge osservando

    \[ \lim_{x \to 0^+} \frac{x-1}{x} = - \infty, \]

perciò sia x^2 che il termine esponenziale tendono a zero; in particolare, si ha:

    \[ \lim_{x\to 0^+}f_1(x)=0\cdot \underbrace{e^{-\infty}}_{=0}=0. \]

La funzione non ammette asintoti verticali e, al contrario, presenta una discontinuità eliminabile in x=0 (ponendo f(0)=0). Abbiamo poi per la derivata prima

    \[ f_1'(x)=2x e^{\frac{x-1}{x}}+x^2 e^{\frac{x-1}{x}}\cdot\frac{x-x+1}{x^2}=(2x+1) e^{\frac{x-1}{x}}. \]

Nel porre f_1'(x) = 0 dobbiamo ricordarci che stiamo lavorando solo sulla semiretta positiva, quindi ogni soluzione è sottoposta all’ulteriore vincolo x>0. Nello specifico, si ha

    \[ 	2x e^{\frac{x-1}{x}}+x^2 e^{\frac{x-1}{x}}\cdot\frac{x-x+1}{x^2}=(2x+1) e^{\frac{x-1}{x}} = 0 \iff x=-\frac12, \]

e questa non è accettabile dato che non soddisfa la condizione x>0. Ne deduciamo che

    \[\quad\]

  • f'(x) > 0 ed f crescente in (0,+\infty);
  •  

  • f'(x) < 0 ed f decrescente in (-\infty, 0);

dato che, come menzionato in altri esercizi, f pari implica f' dispari. Ne deduciamo che l’origine O=(0,0) è un estremo inferiore della funzione. Infine, la derivata seconda è

    \[ f_1''(x)=2 e^{\frac{x-1}{x}}+(2x+1) e^{\frac{x-1}{x}}\cdot\frac{1}{x^2}=\frac{2x^2+2x+1}{x^2}\cdot e^{\frac{x-1}{x}} \]

In questo caso, abbiamo una funzione sempre positiva e, per simmetria, la f(x) risulta convessa nel suo dominio. Il grafico è riportato in figura.

    \[\quad\]

    \[\quad\]

    \[\quad\]

Rendered by QuickLaTeX.com

Soluzione punto 3.

La funzione

    \[ f(x) = \sin x e^{| \sin x|} \]

è definita su tutto \mathbb R e risulta periodica di periodo T=2\pi. Restringiamo dunque lo studio all’intervallo [0,2\pi). Sviluppando il modulo, ovvero

    \[ |\sin x| = \begin{cases} \sin x & \text{se } x \in [0,\pi) \\ - \sin x & \text{se } x \in [\pi,2\pi). \end{cases} \]

In particolare, possiamo riscrivere la funzione come segue:

    \[ f(x) = \begin{cases} \sin x\ e^{\sin x} & \text{se } x \in [0,\pi) \\ \sin x\ e^{-\sin x} & \text{se } x \in [\pi,2\pi). \end{cases} \]

La funzione è periodica e definita ovunque, perciò non ci sono asintoti di alcun tipo. La funzione, ristretta all’intervallo scelto in precedenza, si interseca con gli assi nei punti

    \[ O=(0,0)\quad \text{e} \quad A=(\pi,0). \]

La derivata prima è data da

    \[ \begin{aligned} & x \in [0,\pi) \implies f'(x)=\cos x\ e^{\sin x}+\sin x\ \cos x\ e^{\sin x}=\cos x(1+\sin x)e^{\sin x}, \\ & x \in [\pi,2\pi) \implies f'(x)=\cos x\ e^{-\sin x}-\sin x\ \cos x\ e^{-\sin x}=\cos x(1-\sin x)e^{-\sin x}. \end{aligned}\]

Se denotiamo con f_1' ed f_2' i due rami della funzione derivata, osserviamo che

    \[f_1'(x) = 0 \iff \cos x=0 \text{ e } \sin x=-1\iff x=\frac{\pi}{2},\]

ed analogamente

    \[f_2'(x)=0 \iff \cos x=0 \text{ e } \sin x=1\iff x=\frac{3\pi}{2}.\]

Si verifica che f_1'(x)>0 per x\in(0,\pi/2) e f'_2(x)>0 per x\in (3\pi/2,2\pi), da cui segue che la funzione cresce su (0,\pi/2)\cup(3\pi/2,2\pi) ed ammette un massimo assoluto in M=(\pi/2,e) ed un minimo assoluto in m=(3\pi/2,-e). Per le derivate seconde si ha poi

    \[ \begin{aligned} & f''_1(x)=-\sin x(1+\sin x)e^{\sin x}+\cos^2 x\ e^{\sin x}+\cos^2 x(1+\sin x) e^{\sin x}, \\ & f''_2(x)=-\sin x(1-\sin x)e^{-\sin x}-\cos^2 x\ e^{-\sin x}-\cos^2 x(1-\sin x) e^{-\sin x}. \end{aligned}\]

Utilizzando al formula fondamentale della trigonometria \sin^2 x + \cos^2 x = 1, possiamo riscriverle come segue:

    \[ \begin{aligned}  	& f''_1(x)=-(\sin^3 x+3\sin^2 x-2) e^{\sin x}, \\ & f''_2(x)=-(\sin^3 x-3\sin^2 x+2) e^{-\sin x}. \end{aligned}\]

Per determinare gli zeri di tali derivate, poniamo t=\sin x e consideriamo i risultanti polinomi di terzo grado:

    \[p_1(t)=t^3+3t^2-2,\qquad p_2(t)=t^3-3t^2+2.\]

Notiamo che il primo è definito per t \in (0,1) ed il secondo per t \in (-1,0) e che, per quanto riguarda gli zeri delle derivate, si può dimenticare l’esponenziale dato che è sempre positivo. Si verifica che

    \[p_1(0)=-2,\quad p_1(1)=2,\quad q_1(-1)=-2,\quad q_1(0)=2,\]

per cui possiamo applicare il teorema di esistenza degli zeri secondo cui p_1 e p_2 si annullano almeno una volta nei rispettivi domini di definizione. Inoltre, si ha

    \[p_1'(t)=3t^2+6t=3t(t+2)\geq 0,\qquad p_2'(t)=3t^2-6t=3t(t-2)\leq 0,\]

perciò i due polinomi sono monotoni e, di conseguenza, ammettono un unico zero nei rispettivi domini di definizione. Indichiamo tali punti al modo seguente

    \[t_1\in(0,1),\qquad t_2\in(-1,0).\]

A questo punto, possiamo tornare nella variabile originale e risolvere

    \[ \sin x = t_i \quad \text{per } i = 1,2,\]

ottenendo le seguenti soluzioni:

    \[ \begin{aligned} & x=\alpha,\quad x=\pi-\alpha,\qquad \alpha=\arcsin t_1\in(0,\pi/2), \\ & x=\beta,\quad x=3\pi-\beta,\qquad \beta=\arcsin t_2\in(\pi,3\pi/2), \end{aligned}\]

i quali risultano punti di flesso. La funzione f è pertanto convessa su (0,\alpha)\cup(\pi-\alpha,\pi)\cup(\beta,3\pi-\beta) e concava su (\alpha,\pi-\alpha)\cup(\pi,\beta)\cup(3\pi-\beta,2\pi), da cui segue che ci sono punti di flesso anche in x=0 e x=\pi, ovvero in O ed A. Il grafico è riportato in figura.

    \[\quad\]

    \[\quad\]

    \[\quad\]

Rendered by QuickLaTeX.com

Soluzione punto 4.

La funzione

    \[ f(x) = \arctan \frac{\log x -1}{\log x + 1} + \log(\log^2 x + 1) \]

è definita per

    \[ \begin{cases} x > 0 \\ \log (x) + 1 \neq 0 \\ \log^2 (x) + 1 > 0 \end{cases}\]

Il logaritmo è diverso da -1 quando l’argomento è diverso da 1/e, mentre \log^2 x è sempre positivo e dunque maggiore di -1. Il dominio risulta essere:

    \[D=\left(0,\frac{1}{e}\right)\cup\left(\frac{1}{e},+\infty\right).\]

Facciamo i limiti agli estremi del dominio. Per x \to + \infty osserviamo che

    \[ \lim_{x\to+\infty}f(x)=\lim_{x\to+\infty}\left(\arctan\frac{\log x-1}{\log x +1}+\log(\log^2 x+1)\right)=+\infty, \]

dato che il primo fattore tende ad \arctan 1 = \pi/4 e il secondo a + \infty. Inoltre, si ha

    \[ \lim_{x\to+\infty}\frac{f(x)}{x}=\lim_{x \to +\infty}\frac1x \left(\arctan\frac{\log x-1}{\log x +1}+\log(\log^2 x+1)\right)=0 \]

dato che il denominatore va a infinito più velocemente del numeratore; in particolare, la funzione non presenta asintoti obliqui od orizzontali.

Infine, ponendo t = \log x abbiamo che x \to 0^+ corrisponde a t \to - \infty da cui:

    \[\lim_{x\rightarrow 0^+}f(x)=\lim_{t\rightarrow-\infty}\left(\arctan\frac{t-1}{t+1}+\log(t^2+1)\right)=+\infty,\]

per cui la retta x=0 è un asintoto verticale. Si ha poi

    \[\lim_{x\rightarrow(1/e)^{\pm}}f(x)=\arctan(\mp\infty)+\log 2=\mp\frac{\pi}{2}+\log 2,\]

perciò la funzione in x=1/e ha una discontinuità di prima specie con salto -\pi. Per la derivata prima, posto g(x)=\log x, la funzione si scrive

    \[ f(x)=\arctan\frac{g-1}{g+1}+\log(g^2+1). \]

Derivando si ha allora

    \[ \begin{aligned}  	f'(x) & = \frac{1}{1+\left(\frac{g-1}{g+1}\right)^2}\cdot\frac{g+1-g+1}{(g+1)^2}\cdot g'+\frac{2g g'}{g^2+1} \\ & =g'\left[\frac{(g+1)^2}{g^2+2g+1+g^2-2g+1}\cdot\frac{2}{(g+1)^2}+\frac{2g}{g^2+1}\right]=g'\cdot\frac{1+2g}{g^2+1}, \end{aligned} \]

e sostituendo g = \log x si arriva alla formula:

    \[ f'(x)=\frac{1+2\log x}{x(\log^2 x+1)}. \]

Tale derivata si annulla quando

    \[ 2 \log x + 1 = 0 \iff \log x = -\frac12 \iff x = \frac{1}{\sqrt{e}} \]

e, dato che f'(x)>0 su (1/\sqrt{e},+\infty), la funzione cresce su tale intervallo ed ammette un minimo (assoluto) nel punto

    \[ m= \left(\frac{1}{\sqrt{e}},\arctan(-3)+\log(5/4)\right). \]

Per la derivata seconda si ha

    \[\begin{aligned} f''(x)& =\frac{\displaystyle\frac{2}{x}\cdot x(\log^2 x+1)-(1+2\log x)\cdot\left[\log^2 x+1+x\frac{2\log x}{x}\right]}{x^2(\log^2 x+1)^2} \\ & =\frac{2\log^2 x+2-(1+2\log x)(\log^2 x+2\log x+1)}{x^2(\log^2 x+1)^2} \\ & = -\frac{2\log^3 x+3\log^2 x+4\log x-1}{x^2(\log^2 x+1)^2}. \end{aligned}\]

Per determinare gli zeri di questa derivata, poniamo ancora una volta t=\log x e consideriamo il polinomio associato al numeratore:

    \[ p(t)=2t^3+3t^2+4t-1 \]

definito in (-\infty,\infty). I limiti ad infinito sono

    \[ \lim_{t\to\pm\infty}p(t)=\pm\infty, \]

e la derivata è sempre positiva

    \[p'(t)=6t^2+6t+4=2(3t^2+3t+2)>0, \]

perciò per il teorema di esistenza degli zeri esiste un’unica soluzione a p(t)=0. Dato che p(0)=-1 e p(1)=8, possiamo restringerci all’intervallo (0,1). Denotandola t_0\in(0,1), si ha

    \[\alpha=e^{t_0}\in(1,e)\]

soluzione unica di f''(x)=0. Concludiamo che f''(x)>0 per x\in(0,1/e)\cup(1/e,\alpha), dove la funzione risulta convessa. Infine c’è un flesso nel punto F=(\alpha,f(\alpha)).

Il grafico è riportato nella figura che segue:

    \[\quad\]

    \[\quad\]

    \[\quad\]

Rendered by QuickLaTeX.com


 
 

Riferimenti bibliografici

[1] Stewart, James. Calculus: Early Transcendentals. 8th ed., Cengage Learning, 2015.
Chapter 4: Applications of Derivatives.

[2] Thomas, George B., Weir, Maurice D., and Hass, Joel. Thomas’ Calculus. 14th ed., Pearson, 2017.
Chapter 4: Applications of Derivatives.

[3] Larson, Ron, and Edwards, Bruce H. Calculus. 11th ed., Cengage Learning, 2017.
Chapter 4: Applications of Differentiation.

[4] OpenStax. Calculus Volume 1. OpenStax, 2016.
Chapter 4: Applications of Derivatives.

[5] Spivak, Michael. Calculus. 4th ed., Publish or Perish, 2008.

[6] Lang, Serge. A First Course in Calculus. 5th ed., Springer, 2006.
Chapter 6: Applications of the Derivative.

[7] Qui Si Risolve, Funzioni elementari.

[8] Qui Si Risolve, Funzioni elementari – volume 1.

[9] Qui Si Risolve, Funzioni elementari – volume 2.

[10] Qui Si Risolve, Funzioni goniometriche.

[11] Qui Si Risolve, La teoria delle funzioni continue.

[12] Qui Si Risolve, Teoria sui limiti.

[13] Qui Si Risolve, I teoremi di de l’Hôpital.

[14] Qui Si Risolve, Polinomi di Taylor nei limiti: istruzioni per l’uso.

[15] Qui Si Risolve, Espansione di Taylor: teoria, esempi e applicazioni pratiche.

[16] Qui Si Risolve, Funzioni convesse.

[17] Qui Si Risolve, Teoria derivate.

 
 

Tutta la teoria di analisi matematica

Leggi...

  1. Teoria Insiemi
  2. Il metodo della diagonale di Cantor
  3. Logica elementare
  4. Densità dei numeri razionali nei numeri reali
  5. Insiemi Numerici \left(\mathbb{N},\, \mathbb{Z},\, \mathbb{Q}\right)
  6. Il principio di induzione
  7. Gli assiomi di Peano
  8. L’insieme dei numeri reali: costruzione e applicazioni
  9. Concetti Fondamentali della Retta Reale: Sintesi Teorica
  10. Costruzioni alternative di \mathbb{R}
  11. Binomio di Newton
  12. Spazi metrici, un’introduzione
  13. Disuguaglianza di Bernoulli
  14. Disuguaglianza triangolare
  15. Teoria sulle funzioni
  16. Funzioni elementari: algebriche, esponenziali e logaritmiche
  17. Funzioni elementari: trigonometriche e iperboliche
  18. Funzioni goniometriche: la guida essenziale
  19. Teorema di Bolzano-Weierstrass per le successioni
  20. Criterio del rapporto per le successioni
  21. Definizione e proprietà del numero di Nepero
  22. Limite di una successione monotona
  23. Successioni di Cauchy
  24. Il teorema ponte
  25. Teoria sui limiti
  26. Simboli di Landau
  27. Funzioni continue – Teoria
  28. Il teorema di Weierstrass
  29. Il teorema dei valori intermedi
  30. Il teorema della permanenza del segno
  31. Il teorema di Heine-Cantor
  32. Il teorema di esistenza degli zeri
  33. Il metodo di bisezione
  34. Teorema ponte versione per le funzioni continue
  35. Discontinuità di funzioni monotone
  36. Continuità della funzione inversa
  37. Teorema delle contrazioni o Teorema di punto fisso di Banach-Caccioppoli
  38. Teoria sulle derivate
  39. Calcolo delle derivate: la guida pratica
  40. Teoria sulle funzioni convesse
  41. Il teorema di Darboux
  42. I teoremi di de l’Hôpital
  43. Teorema di Fermat
  44. Teoremi di Rolle e Lagrange
  45. Il teorema di Cauchy
  46. Espansione di Taylor: teoria, esempi e applicazioni pratiche
  47. Polinomi di Taylor nei limiti: istruzioni per l’uso
  48. Integrali definiti e indefiniti
  49. Teorema fondamentale del calcolo integrale (approfondimento)
  50. Integrali ricorsivi
  51. Formule del trapezio, rettangolo e Cavalieri-Simpson
  52. Teoria sugli integrali impropri
  53. Funzioni integrali – Teoria
  54. Introduzione ai numeri complessi – Volume 1 (per un corso di ingegneria — versione semplificata)
  55. Introduzione ai numeri complessi – Volume 1 (per un corso di matematica o fisica)
  56. Serie numeriche: la guida completa
  57. Successioni di funzioni – Teoria
  58. Teoremi sulle successioni di funzioni
    1. 58a. Criterio di Cauchy per la convergenza uniforme
    2. 58b. Limite uniforme di funzioni continue
    3. 58c. Passaggio al limite sotto il segno di integrale
    4. 58d. Limite uniforme di funzioni derivabili
    5. 58e. Piccolo teorema del Dini
    6. 58f. Procedura diagonale e teorema di Ascoli-Arzela
  59. Serie di funzioni – Teoria
  60. Serie di potenze – Teoria
  61. Serie di Fourier – Teoria e applicazioni
  62. Integrali multipli — Parte 1 (teoria)
  63. Integrali multipli — Parte 2 (teoria e esercizi misti)
  64. Regola della Catena — Teoria ed esempi.
  65. Jacobiano associato al cambiamento di coordinate sferiche
  66. Guida ai Massimi e Minimi: Tecniche e Teoria nelle Funzioni Multivariabili
  67. Operatore di Laplace o Laplaciano
  68. Teoria equazioni differenziali
  69. Equazione di Eulero
  70. Teoria ed esercizi sulla funzione Gamma di Eulero
  71. Teoria ed esercizi sulla funzione Beta
  72. Approfondimento numeri complessi
  73. Diverse formulazioni dell’assioma di completezza
  74. Numeri di Delannoy centrali
  75. Esercizi avanzati analisi

 
 

Tutte le cartelle di Analisi Matematica

Leggi...

  1. Prerequisiti di Analisi
    1. Ripasso algebra biennio liceo
    2. Ripasso geometria analitica
    3. Ripasso goniometria e trigonometria
    4. Errori tipici da evitare
    5. Insiemi numerici N,Z,Q,R
    6. Funzioni elementari
    7. Logica elementare
    8. Insiemi
  2. Successioni
    1. Teoria sulle Successioni
    2. Estremo superiore e inferiore
    3. Limiti base
    4. Forme indeterminate
    5. Limiti notevoli
    6. Esercizi misti Successioni
    7. Successioni per ricorrenza
  3. Funzioni
    1. Teoria sulle funzioni
    2. Verifica del limite in funzioni
    3. Limite base in funzioni
    4. Forme indeterminate in funzioni
    5. Limiti notevoli in funzioni
    6. Calcolo asintoti
    7. Studio di funzione senza derivate
    8. Dominio di una funzione
    9. Esercizi misti Funzioni
    10. Esercizi misti sui Limiti
  4. Funzioni continue-lipschitziane-holderiane
    1. Teoria sulle Funzioni continue-lipschitziane-holderiane
    2. Continuità delle funzioni
    3. Continuità uniforme
    4. Teorema degli zeri
    5. Esercizi sul teorema di Weierstrass senza l’uso delle derivate
  5. Calcolo differenziale
    1. Derivate
    2. Calcolo delle derivate
    3. Retta tangente nel calcolo differenziale
    4. Punti di non derivabilità nel calcolo differenziale
    5. Esercizi sul teorema di Weierstrass con l’uso delle derivate
    6. Studio di funzione completo nel calcolo differenziale
    7. Esercizi teorici nel calcolo differenziale
    8. Metodo di bisezione
    9. Metodo di Newton
  6. Teoremi del calcolo differenziale
    1. Teoria sui Teoremi del calcolo differenziale
    2. Teorema di Rolle
    3. Teorema di Lagrange
    4. Teorema di Cauchy
    5. Teorema di De L’Hôpital
  7. Calcolo integrale
    1. Integrale di Riemann
    2. Integrali immediati
    3. Integrale di funzione composta
    4. Integrali per sostituzione
    5. Integrali per parti
    6. Integrali di funzione razionale
    7. Calcolo delle aree
    8. Metodo dei rettangoli e dei trapezi
    9. Esercizi Misti Integrali Indefiniti
    10. Esercizi Misti Integrali Definiti
  8. Integrali impropri
    1. Teoria Integrali impropri
    2. Carattere di un integrale improprio
    3. Calcolo di un integrale improprio
  9. Espansione di Taylor
    1. Teoria Espansione di Taylor
    2. Limiti di funzione con Taylor
    3. Limiti di successione con Taylor
    4. Stime del resto
  10. Funzioni integrali (Approfondimento)
    1. Teoria Funzioni integrali (Approfondimento)
    2. Studio di funzione integrale
    3. Limiti con Taylor e De L’Hôpital
    4. Derivazione di integrali parametrici (Tecnica di Feynmann)
  11. Numeri Complessi
    1. Teoria Numeri complessi
    2. Espressioni con i numeri complessi
    3. Radice di un numero complesso
    4. Equazioni con i numeri complessi
    5. Disequazioni con i numeri complessi
    6. Esercizi misti Numeri complessi
  12. Serie numeriche
    1. Teoria Serie numeriche
    2. Esercizi Serie a termini positivi
    3. Esercizi Serie a termini di segno variabile
    4. Esercizi Serie geometriche e telescopiche
  13. Successioni di funzioni
    1. Teoria Successioni di funzioni
    2. Esercizi Successioni di funzioni
  14. Serie di funzioni
    1. Teoria Serie di funzioni
    2. Esercizi Serie di funzioni
  15. Serie di potenze
    1. Teoria Serie di potenze
    2. Esercizi Serie di potenze
  16. Serie di Fourier
    1. Teoria Serie di Fourier
    2. Esercizi Serie di Fourier
  17. Trasformata di Fourier
    1. Teoria Trasformata di Fourier
    2. Esercizi Trasformata di Fourier
  18. Funzioni di più variabili
    1. Teoria Funzioni di più variabili
    2. Massimi e minimi liberi e vincolati
    3. Limiti in due variabili
    4. Integrali doppi
    5. Integrali tripli
    6. Integrali di linea di prima specie
    7. Integrali di linea di seconda specie
    8. Forme differenziali e campi vettoriali
    9. Teorema di Gauss-Green
    10. Integrali di superficie
    11. Flusso di un campo vettoriale
    12. Teorema di Stokes
    13. Teorema della divergenza
    14. Campi solenoidali
    15. Teorema del Dini
  19. Equazioni differenziali lineari e non lineari
    1. Teoria equazioni differenziali lineari e non lineari
    2. Equazioni differenziali lineari e non lineari del primo ordine omogenee
  20. Equazioni differenziali lineari
    1. Del primo ordine non omogenee
    2. Di ordine superiore al primo,a coefficienti costanti,omogenee
    3. Di ordine superiore al primo,a coefficienti costanti,non omogenee
    4. Di Eulero,di Bernoulli,di Clairaut,di Lagrange e di Abel
    5. Non omogenee avente per omogenea associata un’equazione di Eulero
    6. Sistemi di EDO
  21. Equazioni differenziali non lineari
    1. A variabili separabiliO
    2. A secondo membro omogeneo
    3. Del tipo y’=y(ax+by+c)
    4. Del tipo y’=y(ax+by+c)/(a’x+b’y+c’)
    5. Equazioni differenziali esatte
    6. Mancanti delle variabili x e y
    7. Cenni sullo studio di un’assegnata equazione differenziale non lineare
    8. Di Riccati
    9. Cambi di variabile: simmetrie di Lie
  22. Analisi complessa
    1. Fondamenti
    2. Funzioni olomorfe
    3. Integrale di Cauchy e applicazioni
    4. Teorema della curva di Jordan e teorema fondamentale dell’Algebra
    5. Teorema di inversione di Lagrange
    6. Teorema dei Residui
    7. Funzioni meromorfe
    8. Prodotti infiniti e prodotti di Weierstrass
    9. Continuazione analitica e topologia
    10. Teoremi di rigidità di funzioni olomorfe
    11. Trasformata di Mellin
  23. Equazioni alle derivate parziali
    1. Equazioni del primo ordine
    2. Equazioni del secondo ordine lineari
    3. Equazioni non-lineari
    4. Sistemi di PDE
  24. Funzioni speciali
    1. Funzione Gamma di Eulero
    2. Funzioni Beta,Digamma,Trigamma
    3. Integrali ellittici
    4. Funzioni di Bessel
    5. Funzione zeta di Riemann e funzioni L di Dirichlet
    6. Funzione polilogaritmo
    7. Funzioni ipergeometriche
  25. Analisi funzionale
    1. Misura e integrale di Lebesgue
    2. Spazi Lp,teoremi di completezza e compattezza
    3. Spazi di Hilbert,serie e trasformata di Fourier
    4. Teoria e pratica dei polinomi ortogonali
    5. Spazi di Sobolev
  26. Complementi
    1. Curiosità e approfondimenti
    2. Compiti di analisi
    3. Esercizi avanzati analisi
  27. Funzioni Convesse

 
 

Tutti gli esercizi di geometria

In questa sezione vengono raccolti molti altri esercizi che coprono tutti gli argomenti di geometria proposti all’interno del sito con lo scopo di offrire al lettore la possibilità di approfondire e rinforzare le proprie competenze inerenti a tali argomenti.

Strutture algebriche.





 
 

Risorse didattiche aggiuntive per approfondire la matematica

Leggi...

  • Math Stack Exchange – Parte della rete Stack Exchange, questo sito è un forum di domande e risposte specificamente dedicato alla matematica. È una delle piattaforme più popolari per discutere e risolvere problemi matematici di vario livello, dall’elementare all’avanzato.
  • Art of Problem Solving (AoPS) – Questo sito è molto noto tra gli studenti di matematica di livello avanzato e i partecipanti a competizioni matematiche. Offre forum, corsi online, e risorse educative su una vasta gamma di argomenti.
  • MathOverflow – Questo sito è destinato a matematici professionisti e ricercatori. È una piattaforma per domande di ricerca avanzata in matematica. È strettamente legato a Math Stack Exchange ma è orientato a un pubblico con una formazione più avanzata.
  • PlanetMath – Una comunità collaborativa di matematici che crea e cura articoli enciclopedici e altre risorse di matematica. È simile a Wikipedia, ma focalizzata esclusivamente sulla matematica.
  • Wolfram MathWorld – Una delle risorse online più complete per la matematica. Contiene migliaia di articoli su argomenti di matematica, creati e curati da esperti. Sebbene non sia un forum, è una risorsa eccellente per la teoria matematica.
  • The Math Forum – Un sito storico che offre un’ampia gamma di risorse, inclusi forum di discussione, articoli e risorse educative. Sebbene alcune parti del sito siano state integrate con altri servizi, come NCTM, rimane una risorsa preziosa per la comunità educativa.
  • Stack Overflow (sezione matematica) – Sebbene Stack Overflow sia principalmente noto per la programmazione, ci sono anche discussioni rilevanti di matematica applicata, specialmente nel contesto della scienza dei dati, statistica, e algoritmi.
  • Reddit (r/Math) – Un subreddit popolare dove si possono trovare discussioni su una vasta gamma di argomenti matematici. È meno formale rispetto ai siti di domande e risposte come Math Stack Exchange, ma ha una comunità attiva e molte discussioni interessanti.
  • Brilliant.org – Offre corsi interattivi e problemi di matematica e scienza. È particolarmente utile per chi vuole allenare le proprie capacità di problem solving in matematica.
  • Khan Academy – Una risorsa educativa globale con lezioni video, esercizi interattivi e articoli su una vasta gamma di argomenti di matematica, dalla scuola elementare all’università.






Document









Document